Vous êtes sur la page 1sur 108

NBME FORM 2 Block 1

1.) A 50-year-old man with a 20-year history of type 2 diabetes mellitus has had sensory neuropathy for 2 weeks. Pulses are decreased at the ankle. There is no peripheral edema. His serum glucose levels have been between 150 mg/dL and 200 mg/dL over the past 6 months. Which of the following is the most effective measure to prevent serious foot infections? A) Use of support hose B) Use of well-fitted shoes C) Prophylactic antibiotics D) Daily aspirin and dipyridamole E) Decrease in serum glucose level 2.) A 63-year-old woman comes to the emergency department because of palpitations for 1 hour. She was diagnosed with hyperthyroidism 2 months ago but has not received treatment. Three hours before arrival, she drank two bottles of beer with dinner. Her blood pressure is 90/60 mm Hg, and pulse is 85/min and irregularly irregular. Examination shows a variation in intensity of S1. There is an absence of waves in the venous pulse. An ECG is shown. Which of the following is the most likely cardiac rhythm disturbance? A) Atrial fibrillation B) Atrial flutter C) Multifocal atrial tachycardia D) Paroxysmal atrial tachycardia E) Second-degree atrioventricular block, Wenckebach type F) Sick sinus syndrome G) Sinoatrial exit block, type II H) Torsades de pointes I) Wandering pacemaker J) Wolff-Parkinson-White syndrome 3.) A 5-year-old girl is brought to the physician 30 minutes after being bitten on the forearm by her cat. Examination shows a small puncture wound covered with dried blood. She is at increased risk of infection for which of the following reasons? A) Arm wounds are more susceptible to infection

B) Salivary enzymes in cats augment infection C) It is difficult to cleanse pathogens from this wound D) The Staphylococcus species involved is particularly virulent E) Young children are less able to resist infection than adults 4.) A 4-year-old girl is brought to the physician because of pallor and jaundice for 2 days. She had previously been well, although she was treated for jaundice with phototherapy for 2 weeks while a newborn. Her mother and two additional maternal relatives underwent splenectomy during childhood for unknown reasons. Examination of the patient shows jaundice. The spleen tip is palpated 4 cm below the left costal margin. Which of the following blood smear findings is most likely to explain this family's condition ? A) Elliptocytes B) Howell-Jolly bodies C) Schistocytes D) Sickle cells E) Spherocytes 5.) A sexually active 20-year-old woman has had fever, chills, malaise, and pain of the vulva for 2 days. Examination shows a vulvar pustule that has ulcerated and formed multiple satellite lesions. Nodes are palpated in the inguinal and femoral areas. A smear of fluid from the lesions establishes the diagnosis. Which of the following is the most likely causal organism? A) Chlamydia trachomatis B) Haemophilus ducreyi C) Neisseria gonorrhoeae D) Streptococcus pyogenes (group A) E) Treponema pallidum 6.) A 5-year-old boy is brought to the physician 1 hour after urinating bright red blood. He has been taking ibuprofen since injuring his right flank while wrestling with friends yesterday; he also has been taking penicillin for 3 days for streptococcal pharyngitis. His temperature is 36.7 C (98 F), blood pressure is 90/48mm Hg, pulse is 108/min, and respirations are 18/min. Examination shows purple ecchymoses over the shins and right flank; there is tenderness of the right costovertebral area. The abdomen is nontender. Genital examination shows no abnormalities. There is no edema. Urinalysis shows gross blood; microscopic examination shows 510 leukocytes/hpf and erythrocytes that are too numerous to count. Which of the following is the most likely explanation for this patient's hematuria? A) Acute pyelonephritis B) Ibuprofen-induced acute papillary necrosis

C) Post-streptococcal glomerulonephritis D) Rhabdomyolysis E) Traumatic injury to the kidney 7.) A 16-year-old girl comes to the physician for her first prenatal visit at 12 weeks' gestation. She has not had any immunizations since the age of 5 years. She has received the following immunizations at the recommended ages: 5 3 1 4 Diphtheria-tetanus-pertussis Hepatitis B Measles-mumps-rubella Oral poliovirus

Examination shows no abnormalities. Which of the following is the most appropriate immunization to administer at this visit? A) Diphtheria-tetanus-pertussis B) Diphtheria (child)-tetanus toxoid C) Diphtheria (adult)-tetanus toxoid D) Haemophilus influenzae type b E) Hepatitis B F) Inactivated poliovirus 8.) A 12-year-old boy is brought to the physician because of temperatures to 40.2 C (104.4 F), lethargy, and a stiff neck for 2 days. A diagnosis of pneumococcal meningitis is made, and the child is admitted to the hospital for antibiotic therapy. Serum studies 24 hours after admission show: Na+ 117 mEq/L Cl 89 mEq/L HCO3 25 mEq/L Which of the following is the most likely explanation for these findings? A) Excessive cortisol release B) Hyperaldosteronism C) Inappropriate secretion of ADH (vasopressin) D) Increased production of leukotriene C4 9. )An asymptomatic 57-year-old man with a 3-year history of type 2 diabetes mellitus comes for a routine follow-up visit. Examination shows no abnormalities. Serum studies show: Aspartate aminotransferase (AST, GOT) 76 U/L Alanine aminotransferase (ALT, GPT) 86 U/L Iron 260 g/dL Total iron-binding capacity 300 g/dL (N=250450) Ferritin 1200 ng/mL Antinuclear antibody negative

Serologic testing for hepatitis is negative. Which of the following is the most appropriate next step in management? A) Corticosteroid therapy B) Interferon therapy C) Penicillamine therapy D) Chronic phlebotomy E) No therapy indicated 10.) A 26-year-old man has had anxiety and insomnia since he lost his job 1 month ago. He also has had palpitations, daily headaches, and flatulence. He has no history of psychiatric disorders. His mother has a history of bipolar disorder, mixed, and his father has alcoholism; his younger sister had a history of drug abuse but has been abstinent from drugs for 3 years. Which of the following is the most likely diagnosis? A) Adjustment disorder B) Bipolar disorder, depressed C) Generalized anxiety disorder D) Major depressive disorder E) Post-traumatic stress disorder

11.) A previously healthy 32-year-old man comes to the emergency department because of a 12-hour history of fatigue and shortness of breath at rest. He takes no medications. His temperature is 7 C (98.6 F), blood pressure is 96/58 mm Hg, pulse is 110/min, and respirations are 22/min. Examination shows scleral icterus and conjunctival pallor. There is no hepatosplenomegaly. Laboratory Studies show: Hemoglobin 6.2 g/dL Serum Leukocyte count 8800/mm3 Bilirubin, total 8.5 mg/dL Reticulocyte count 16% Direct 1.5 mg/dL Platelet count 245,000/mm3 Lactate dehydrogenase 1260 U/L A) Acute toxic hepatitis B) Alcoholic hepatitis C) Cholangiocarcinoma D) Chronic idiopathic cirrhosis E) Common bile duct calculus F) Congestive hepatitis G) Gallstone pancreatitis

H) Gilbert's syndrome I) Infectious hepatitis J) Intravascular hemolysis K) Primary biliary cirrhosis 12.) A 24-year-old third-year medical student comes to the student health clinic for evaluation of jaundice which he noticed this morning. He has had no abdominal pain, itching, or weight change. He takes no medications. He describes recent anxiety over anticipation of the upcoming Step 2 examination. His temperature is 37 C (98.6 F), blood pressure is 120/80 mm Hg, pulse is 76/min, and respirations are 12/min. Examination shows no abnormalities except for scleral icterus. Laboratory studies show: Hemoglobin 15 g/dL Serum Leukocyte count 7000/mm3 Alkaline phosphatase 90U/L Platelet count 240,000/mm3 Aspartate aminotransferase Serum (AST, GOT) 23 U/L Bilirubin, total 3.5 mg/dL Alanine aminotransferase Direct 0.7 mg/dL (ALT, GPT) 27 U/L A) Acute toxic hepatitis B) Alcoholic hepatitis C) Cholangiocarcinoma D) Chronic idiopathic cirrhosis E) Common bile duct calculus F) Congestive hepatitis G) Gallstone pancreatitis H) Gilbert's syndrome I) Infectious hepatitis J) Intravascular hemolysis K) Primary biliary cirrhosis 13.) A 37-year-old woman with a 2-year history of hypertension comes for a follow-up examination; her hypertension has worsened despite treatment with a low-sodium diet and a -adrenergic blocking agent. She has a history of rheumatic fever and Graves' disease treated with 131I. Her blood pressure is 160/106 mm Hg, and pulse is 80/min. Serum studies show: Na+ 135 mEq/L Cl 100 mEq/L K+ 3.4 mEq/L HCO3 26 mEq/L Urea nitrogen (BUN) 12 mg/dL Creatinine 0.8 mg/dL

Urinalysis is within normal limits. Renal ultrasonography shows a left kidney with a markedly irregular contour; it is 2.8 cm smaller than the right. A) Adrenal cortex B) Adrenal medulla C) Aorta D) Renal arteries E) Renal glomeruli F) Thyroid gland 14.) A 27-year-old woman comes to the physician because of muscle weakness and cramps for 2 weeks. She has been taking a -adrenergic blocking agent for hypertension for 2 years. She had chronic lymphocytic thyroiditis (Hashimoto's disease) 1 year ago. Her blood pressure is 160/108 mm Hg, and pulse is 60/min. Serum studies show: Na+ 140 mEq/L Cl 110 mEq/L K+ 2.2 mEq/L HCO3 30 mEq/L Urea nitrogen (BUN) 20 mg/dL Creatinine 1 mg/dL Magnetic resonance angiography of the abdomen shows normal findings. A) Adrenal cortex B) Adrenal medulla C) Aorta D) Renal arteries E) Renal glomeruli F) Thyroid gland 15.) A 24-year-old man is hospitalized for treatment of a posterior dislocation of the right knee sustained in a motorcycle collision. Six hours after closed reduction, previously present distal pulses in his foot are absent, but the foot has remained warm. Which of the following is the most appropriate next step in management? A) Elevation of the limb and observation for 24hours B) Nitroprusside therapy C) Sympathetic block D) Femoral arteriography E) Embolectomy with a Fogarty catheter through aproximal arteriotomy D) Inflammation of the origin of the plantar fascia

16.) A healthy 4-year-old girl is brought for a well-child examination. A grade 2/6 systolic ejection murmur is heard along the upper left sternal border. S2 is widely split and does not vary with respiration. A soft mid-diastolic murmur is heard along the lower left sternal border. Examination shows no other abnormalities. Which of the following is the most likely diagnosis? A) Aortic stenosis B) Atrial septal defect C) Coarctation of the aorta D) Mitral valve prolapse E) Patent ductus arteriosus F) Pulmonary stenosis G) Tetralogy of Fallot H) Transposition of the great arteries I) Ventricular septal defect J) Normal heart

18.) A previously healthy 57-year-old woman comes to the physician because of three episodes of blurred vision in the right eye over the past 3 weeks; each episode lasts approximately 5minutes. Retinal examination shows a small refractile body at The bifurcation of a retinal artery. The remainder of the examination shows no abnormalities. Which of the following is the most appropriate next step in diagnosis? A) Cerebral angiography B) Echocardiography C) Electroencephalography D) Duplex scan of the carotid arteries E) MRI of the brain 19.) A 67-year-old woman comes to the physician for her first influenza virus vaccination. She has a history of untreated hypertension. Her blood pressure is 160/100 mm Hg, and pulse is 100/min. Shortly after administration of the influenza virus vaccine, she develops shortness of breath, hives, and angioedema. Which of the following is most likely to have prevented this reaction? A) Inquiry about an egg allergy B) Heterophile agglutination test C) Skin test with histamine reagent

D) -Adrenergic blocking agent therapy E) Amantadine therapy F) Insulin therapy G) Rimantadine therapy

20.) A 7-year-old girl is brought to the physician because of a 2-day history of fever, headache, sore throat, and swollen glands. She does not have a runny nose, congestion, or cough. She has no allergies to medications. Her temperature is 38.6C (101.4 F), blood pressure is 100/60 mm Hg, pulse is 120/min, and respirations are 16/min. Examination shows a swollen, erythematous oropharynx With tonsillar exudates. The anterior cervical lymph nodes are enlarged and tender. No other abnormalities are noted. Which of the following is the most likely causal organism? A) Adenovirus B) Corynebacterium diphtheriae C) Group A streptococcus D) Haemophilus influenzae E) Mycoplasma pneumoniae 21.) A 70-year-old nursing home resident is admitted to the hospital because of progressive obtundation over the past 2 days. He has tachycardia, tachypnea, and hypotension. Bilateral basilar crackles and an S3 gallop are heard on auscultation. Examination shows jugular venous distention and peripheral edema. Swan-Ganz catheterization shows a cardiac index of 1.8 L/min/m2 (N=2.54.2), a mean Pulmonary capillary wedge pressure of 23 mm Hg (N=110), and markedly Increased systemic vascular resistance. Which of the following is the most likely diagnosis? A) Cardiogenic shock B) Hypovolemic shock C) Neurogenic shock D) Septic shock

22.) A 42-year-old man with alcoholism is brought to the emergency department by a friend because of fever and progressive shortness of breath for 12 hours. His friend reports that they were eating leftover chicken and drinking beer earlier in the day when the patient suddenly choked and vomited. Six hours later, he developed sweating, chills, and shortness of breath. He has not had any subsequent nausea, vomiting, hematemesis, or abdominal pain. He has smoked Two packs of cigarettes daily for 22 years and drinks 12 to 18 beers daily. He is in moderate respiratory distress. His temperature is 39.3 C (102.8 F), blood pressure is 90/60 mm Hg, pulse is 120/min, and respirations are 24/min. Examination shows no jugular venous distention. Breath sounds are decreased halfway up the left lung with increased dullness. Cardiac examination shows a normal S1 and S2; no murmurs are heard. There is no abdominal tenderness. Bowel sounds are

hypoactive. X-ray films of the chest show a left pleural effusion and air in the mediastinum. Thoracentesis is performed. Laboratory studies show: Leukocyte count 18,000/mm3 Segmented neutrophils 85% Bands 10% Lymphocytes 5% Serum Protein 6 g/dL Lactate dehydrogenase 200 U/L Pleural fluid Leukocyte count 8000/mm3 Segmented neutrophils 98% Monocytes 2% Protein 4.2 g/dL Amylase 140 U/L Lactate dehydrogenase 180 U/L Gram's stain WBC present Organisms none Which of the following is the most likely diagnosis? A) Congestive heart failure B) Esophageal rupture C) Pancreatitis D) Pericarditis E) Tuberculosis 23.) A 2-year-old girl with tricuspid atresia has increasing respiratory distress for 2 days. She has been recovering uneventfully from an operation 10 days ago to join systemic venous return with pulmonary arterial circulation. Over the past 4 days, she has been weaned off mechanical ventilation, started on oral feedings, and is receiving chest physiotherapy for atelectasis. Her temperature is 37.4 C (99.3 F), blood pressure is 98/64 mm Hg, pulse is 120/min, and respirations are 46/min. Examination shows nasal flaring, grunting, and intercostal retractions. An x-ray film of the chest shows large bilateral pleural effusions. Thoracentesis yields 300 mL of whitish-yellow fluid. The supernatant remains uniformly opaque on centrifugation. Which of the following is the most likely cause of the pleural effusions? A) Chylothorax B) Congestive heart failure C) Empyema D) Pulmonary embolism E) Superior vena cava obstruction 24.) A 23-year-old man is brought to the physician by his mother because he has heard a voice over the past month telling him to hurt himself. His mother says that her son has no friends and is a lifelong loner; since graduating from high school, he has been unable to hold a job. He admits to smoking marijuana occasionally and drinking six beers weekly. Examination shows a poorly groomed man with poor eye contact. He has a flat affect and limited facial expression. He says he has no intention of harming himself or others. Which of the following is the most

appropriate next step in management? A) Schedule a follow-up visit in 4 weeks B) Prescribe oral risperidone and schedule a follow-up visit in 2 weeks C) Admit him to the partial hospital program and prescribe oral lithium carbonate D) Admit him to the psychiatric unit for detoxification E) Admit him to the psychiatric unit and prescribe oral imipramine

25.) An 18-month-old boy is brought for a well-child examination. He was born at 37 weeks' gestation and weighed 2800 g (6 lb 3oz). There were no prenatal or perinatal complications. Developmental history indicates that he is able to run and walk up stairs while holding his mother's hand. He can hold a crayon but does not Scribble spontaneously. He is at the 40th percentile for length and 50th percentile for weight. Examination shows a crude pincer grasp. Which of the following is the most appropriate assessment of fine and gross motor development? Fine Motor Gross Motor Development Development A) Delayed normal B) Normal delayed C) Delayed delayed D) Normal normal

26.) Five months after beginning fluoxetine to treat obsessive-compulsive disorder, a 19-year-old man states that he discontinued his medication 2 months ago because he had begun to worry about taking his medication every day. His initial response to the medication was good. His symptoms have now returned, and his morning ritual of cleaning and grooming consumes so much time that his job is in jeopardy. In addition to education about the nature of his disorder and its treatment, which of the following is the most appropriate next step in management? A) Tell the patient to schedule a return visit as needed B) Offer to change the medication C) Request that the patient's parents superviseadministration of medication D) Monitor the patient's compliance by weekly blood tests E) Begin a trial of cognitive-behavior therapy

27.) A 32-year-old woman comes to the physician because of a 3-month history of increasing pain and stiffness in her wrists, hands, and ankles. During this period, she also has had progressive fatigue and morning stiffness lasting 2 hours. She has a 1-year history of rheumatoid arthritis treated with naproxen. Examination shows

redness, swelling, and warmth over the wrist, hand, and ankle joints bilaterally. There are nontender subcutaneous nodules over the extensor surfaces of both elbows. X-ray films of the hands show diffuse osteopenia and erosions over several of the distal metacarpal bones. Which of the following is the most appropriate pharmacotherapy? A) Add oral cyclophosphamide B) Add oral gold C) Add oral methotrexate D) Add oral penicillamine E ) Switch to oral ibuprofen

28.) A previously healthy 62-year-old man comes to the physician because of a 2-month history of progressive shortness of breath and a mild nonproductive cough. He does not smoke. He worked in a foundry most of his adult life before retiring 2 years ago. Vital signs are within normal limits. Crackles are heard at both lung Bases with no wheezes. Cardiac examination shows an accentuated P2. The remainder of the examination shows no abnormalities. An x-ray film of the chest shows prominent interstitial markings at the lung bases. Echocardiography shows an ejection fraction of 55%. Pulmonary function testing is most likely to show which of the following? A) Decreased FEV1:FVC ratio B) Decreased maximal inspiratory effort C) Decreased total lung capacity D) Increased forced vital capacity E) Normal carbon monoxide diffusion capacity

29.) A 55-year-old man is admitted to the hospital because of progressive shortness of breath for 10 days. He has a history of chronic venous stasis and deep venous thrombosis. He has been treated with warfarin since he had several pulmonary emboli 2 years ago; he takes no other medication. He has smoked two packs of Cigarettes daily for 30 years. He weighs 109 kg (240 lb) and is 165 cm (65 in) tall. Examination shows jugular venous distention. Echocardiography is most likely to show which of the following? A) Left ventricular dilation B) Mitral stenosis C) Pericardial thickening and tamponade D) Right ventricular hypertrophy and dilation E) Tricuspid stenosis 30.) A 32-year-old woman comes to the physician because of weakness of the lower extremities for 2 days. Three years ago, she had pain and partial loss of vision of the right eye; the vision returned to normal after 6 weeks. There is mild pallor of

the right optic disc. She has impaired tandem gait. Babinski's sign is present bilaterally. There is mild spasticity of the lower extremities and mild weakness of the iliopsoas and hamstring muscles. Serum creatine kinase activity is 50 U/L. Which of the following is the most likely diagnosis? A) Amyotrophic lateral sclerosis B) Becker's muscular dystrophy C) Diffuse sensorimotor peripheral neuropathy D) Duchenne's muscular dystrophy E) Hyperkalemic periodic paralysis F) Hypokalemic periodic paralysis G) Multiple sclerosis H) Myasthenia gravis I) Myasthenic (Lambert-Eaton) syndrome J) Myotonic muscular dystrophy K) Polymyositis L) Pontine astrocytoma M) Pontine infarction N) Spinal cord tumor O) Transverse myelitis

31.) A healthy 42-year-old man comes to the physician for a life insurance evaluation. He smoked one-half pack of cigarettes daily for 20 years but quit 10 years ago. His father died of a myocardial infarction at the age of 65 years. The patient weighs 93 kg (205lb) and is 178 cm (70 in) tall. His blood pressure is 160/110 mm Hg, pulse is 96/min, and respirations are 16/min. Physical examination, ECG, and an xray film of the chest show no abnormalities. Laboratory studies are within normal limits except for a serum cholesterol level of 206 mg/dL. Which of the following is the greatest risk factor for cerebral infarction in this patient? A) Genetic profile B) History of smoking C) Hypercholesterolemia D) Hypertension E) Obesity 32.) A previously healthy 56-year-old woman comes to the physician because of jaundice and dark urine for 3 weeks. She has a 1-year history of generalized pruritus. She takes no medications. Examination shows jaundice and several ecchymoses over the forearms and thighs. The liver and spleen are enlarged and nontender. Laboratory studies show:

Prothrombin time 18 sec Serum Protein Total 8.5 g/dL Albumin 3.8 g/dL Bilirubin Total 5 mg/dL Direct 2 mg/dL Alkaline phosphatase 150 U/L Alanine aminotransferase (ALT, GPT) 45 U/L Antimitochondrial antibody assay is strongly positive. A CT scan of the abdomen shows hepatosplenomegaly. Endoscopic retrograde cholangiopancreatography shows no abnormalities. Because of her condition, this patient is at greatest risk for which of the following deficiencies? A) Niacin B) Vitamin A C) Vitamin B2 (riboflavin) D) Vitamin B12 (cyanocobalamin) E) Vitamin C 33.) An asymptomatic 52-year-old man comes for a follow-up examination 1 month after he passed renal calculi. He has a history of renal calculi 2 years ago. Serum uric acid and calcium levels and urinary oxalate excretion are within normal limits. Urinary calcium excretion is increased. In order to avoid recurrence of renal calculi, which of the following is the most appropriate pharmacotherapy for this patient? A ) Bicarbonate B ) Calcium lactate C ) Methenamine mandelate D ) Probenecid E ) Thiazide diuretic 34. A 57-year-old woman is brought to the emergency department because of abdominal pain for 12 hours. Over the past 3 hours,

the pain has become severe and generalized. Over the past month, she has had mild upper abdominal discomfort that is relieved by eating. She has a history of recurrent migraines treated with sumatriptan as needed. Her temperature is 38.2 C (100.8 F), blood pressure is 170/95 mm Hg, and pulse is 110/min. Abdominal examination shows mild distention; there is marked rigidity with diffuse tenderness. Bowel sounds are absent. Rectal examination shows no abnormalities; test of the stool for occult blood is negative. Laboratory studies show: Hematocrit 36% Leukocyte count 16,500/mm3 Serum Na+ 145 mEq/L Cl 106 mEq/L K+ 3.8 mEq/L HCO3 19 mEq/L Urea nitrogen (BUN) 32 mg/dL Which of the following is the most appropriate next step in diagnosis? A ) X-ray films of the abdomen while supine and standing B ) Abdominal ultrasonography C ) Upper gastrointestinal series with contrast D ) HIDA scan E ) Fiberoptic endoscopy of the upper gastrointestinal tract

35. An 18-year-old man comes to the physician because of itchy lesions on his penis for 2 weeks and itching around his wrists and ankles for 1 week. He is sexually active and does not consistently use condoms. Examination shows scattered, crusted lesions on the penis and no significant inguinal lymphadenopathy. There are excoriated papules on the wrists and ankles, a few papules between the fingers, and excoriations along the belt line. Which of the following is the

most likely diagnosis? A ) Disseminated primary herpes simplex B ) Eczema C ) Scabies D ) Secondary syphilis E ) Vasculitis 36. An asymptomatic 62-year-old man comes for a follow-up visit. One month ago, he had acute cystitis treated with ciprofloxacin. At his initial visit, a urine culture grew Klebsiella pneumoniae. He has a 2-year history of chronic prostatitis and has had four episodes of cystitis over the past year. His temperature is 36.9 C (98.4 F). Examination, including prostate examination, shows no abnormalities. Which of the following is the most appropriate measure to prevent recurrent episodes of cystitis in this patient? A ) Trimethoprim-sulfamethoxazole prophylaxis B ) Insertion of a Foley catheter C ) Total prostatectomy D ) Transurethral prostatectomy E ) No treatment is available 37. A 32-year-old woman comes for a routine health maintenance examination. Examination shows a bloody discharge from the nipple of her left breast. There is no palpable mass. On questioning, she says that she never noted the discharge and does not perform breast self-examinations. Which of the following is the most likely diagnosis?

A ) Breast abscess B ) Breast cyst C ) Breast hematoma D ) Fibroadenoma E ) Fibrocystic changes of the breast F ) Intraductal adenoma 38. A 35-year-old woman is brought to the physician by her husband because of increasing memory loss and involuntary "dancing" movements over the past 6 weeks. She has had difficulty remembering things, particularly recent events. She has had no disturbances in sleep or appetite. Her medical history is unremarkable. She does not remember her mother who died 25 years ago, but her father told her that her mother had similar symptoms several years before her death. Neurologic examination shows involuntary choreiform movements. She has a childlike affect. She describes her mood as good. On recall testing, she can remember one out of three objects after 3 minutes with distraction; she cannot remember the other objects with prompting. She cannot remember the date of her marriage or her previous address. The most likely cause of these symptoms is a lesion at which of the following locations? A ) Caudate nucleus B ) Parietal lobe C ) Prefrontal lobe D ) Putamen E ) Temporal lobe

39. A 67-year-old man comes to the physician because of insomnia for 2 years. He goes to bed at 11:00 PM after taking a bath but does not fall asleep until midnight. He usually wakes up twice each night: once around 3:00 AM to void and again at 6:00 AM. He stays in bed until his alarm goes off at 7 AM. He is concerned because he used to sleep 8 hours daily. He recently retired from his job as an attorney. He takes no medications. He drinks one beer with lunch 3 days each week and one glass of wine with dinner each evening. He walks 1 mile daily after dinner. Physical examination shows no abnormalities. His mood is neutral, and cognition is intact. Urine toxicology screening is negative. Which of the following is the most likely cause of this patient's symptoms? A ) Advanced sleep phase syndrome B ) Alcohol abuse C ) Delayed sleep phase syndrome D ) Major depressive disorder E ) Melatonin deficiency F ) Poor sleep hygiene G ) Normal aging 40. An asymptomatic 37-year-old woman comes for a follow-up examination 1 year after receiving a renal transplant. Current medications include felodipine, enalapril, cyclosporine, pravastatin, and penicillin G. She appears well. Her temperature is 36.8 C (98.2 F), blood pressure is 160/95 mm Hg, pulse is 80/min, and respirations are 12/min. Examination shows no other abnormalities. Which of the following medications is the most likely cause of this patient's hypertension? A

) Cyclosporine B ) Enalapril C ) Felodipine D ) Penicillin G E ) Pravastatin 41. A 4080-g (9-lb) male newborn is delivered at term to a 32-year-old woman, gravida 2, para 1. Apgar scores are 8 and 9 at 1 and 5 minutes, respectively. Examination in the delivery room shows fracture of the right clavicle. Which of the following is the most likely sequela of this condition? A ) Correction only with casting B ) Correction only with physical therapy C ) Left-hand dominance D ) Permanent nerve damage E ) Spontaneous healing without treatment 42. A 22-year-old man with schizoaffective disorder is brought to the emergency department 2 hours after the sudden onset of neck and back pain. The symptoms began after taking one of his medications for his psychiatric disorder. The dosage was increased yesterday, but he does not recall the names of either of his medications. He has no other history of serious illness. Physical examination shows rigid contraction of the neck and back muscles with arching. On mental status examination, he is alert and cooperative and hears a faint voice that tells him to wash windows. He is oriented to person, place, and time. Which of the following medications is most likely responsible for this patient's symptoms?

A ) Clozapine B ) Haloperidol C ) Lithium carbonate D ) Trazodone E ) Valproic acid 43. A 52-year-old woman, gravida 3, para 3, comes to the physician because of irregular vaginal bleeding over the past 2 months. She has hypertension treated with enalapril and type 2 diabetes mellitus well controlled with diet. Menopause occurred 2 years ago. Her maternal aunt had breast cancer at the age of 70 years. The patient weighs 88 kg (195 lb) and is 160 cm (63 in) tall. Examination shows no other abnormalities. An endometrial biopsy specimen shows adenocarcinoma. Which of the following is the most significant predisposing factor for this patient's endometrial cancer? A ) Heredity B ) Hypertension C ) Parity D ) Type 2 diabetes mellitus E ) Weight 44. A 67-year-old man is brought to the emergency department 2 hours after the onset of weakness and double vision. He has hypertension and hyperlipidemia treated with metoprolol, captopril, and atorvastatin. His blood pressure is 190/106 mm Hg. Neurologic examination shows left-sided facial weakness including the forehead. There is palsy of left conjugate gaze, and the left eye fails to adduct on right gaze.

Vertical eye movements are intact. Muscle strength is 3/5 in the right upper and lower extremities. Deep tendon reflexes are brisk, and Babinski's sign is present on the right. Which of the following is the most likely location of this patient's lesion? A ) Bilateral thalamic B ) Left frontal C ) Left pontine D ) Right caudate E ) Right midbrain 45. Six weeks after spontaneous drainage of an anal abscess, a 32-year-old man has persistent blood-stained purulent fluid on his underwear. He has not had significant anal pain since drainage of the boil. Bowel movements are normal. Which of the following is the most likely diagnosis? A ) Anal fissure B ) Fistula in ano C ) Pruritus ani D ) Thrombosed external hemorrhoids E ) Thrombosed internal hemorrhoids 46. A previously healthy 14-year-old girl is brought to the physician because of a 2-day history of fever and pain and swelling of the right knee. She remembers injuring the knee while playing soccer last week, but she was able to finish the game. She has no history of rash or joint pain. Her sister has inflammatory bowel disease. The patient's temperature is 39 C (102.2 F), blood pressure is 110/80 mm Hg, pulse is

95/min, and respirations are 20/min. Examination of the right knee shows swelling, tenderness, warmth, and erythema; range of motion is limited. Which of the following is the most appropriate next step in management? A ) X-ray film of the right knee B ) Gastrointestinal series with small-bowel follow-through C ) Nuclear scan of the right knee D ) MRI of the right knee E ) Antibiotic therapy F ) Arthrocentesis

block 2 dolly123 - 11/07/06 17:58 #548028

1. A 25-year-old man is brought to the emergency department after being discovered semiconscious and incoherent at home. On arrival, he is stuporous. His blood pressure is 105/70 mm Hg, pulse is 80/min, and respirations are 12/min. Examination shows cool, damp skin. The pupils are pinpoint and react sluggishly to light. Which of the following is the most likely substance taken? A ) Alcohol B ) Barbiturates C ) Cocaine D

) Heroin E ) LSD

2. A 15-year-old girl is brought to the emergency department by her 20-year-old sister because of a 1-week history of fatigue, nausea, and abdominal pain. Menarche was at the age of 12 years, and her last menstrual period was 3 weeks ago. She has not had vaginal discharge. She is sexually active, and she and her partner use condoms inconsistently. Her temperature is 37 C (98.6 F), blood pressure is 110/60 mm Hg, and pulse is 95/min. Abdominal examination shows mild bilateral lower quadrant tenderness. Before a pelvic examination and a pregnancy test can be performed in this patient, consent must be obtained from which of the following? A ) The court B ) The patient C ) The patient's parent D ) The patient's sister E ) No consent is necessary

3. A previously healthy 57-year-old man comes to the physician because of a nonpruritic rash over both legs for 1 week and a low-grade fever for 2 days. He recently returned from a 2-week canoe trip on a river in Minnesota. His temperature is 37.2 C (99 F). Examination shows a 4 x 6-cm, macular, dark pink, ovoid lesion on the right posterior hip with central clearing and a punctate eschar near the center. There are macular ring lesions with central clearing over the medial tibia and anterior thighs of the lower extremities. He has no lymphadenopathy. Which of the following is the most likely causal organism?

A ) Borrelia burgdorferi B ) Brucella melitensis C ) Francisella tularensis D ) Leptospira interrogans E ) Rickettsia rickettsii 4. A 57-year-old woman comes to the physician because of a 2-year history of increasing menstrual flow. She has not had hot flashes, insomnia, or change in bowel or bladder function. Her last menstrual period was 2 weeks ago. Pelvic examination shows a normal-appearing vulva, vagina, and cervix. The uterus is consistent in size with an 8-week gestation. Bimanual examination shows a 4-cm, firm, nontender left ovary. An endometrial biopsy specimen shows atypical complex endometrial hyperplasia. Which of the following is the most likely cause of this patient's hyperplasia? A ) Adrenal adenoma B ) Brenner tumor C ) Carcinoid tumor D ) Granulosa cell tumor E ) Hyperthecosis F ) Islet cell tumor G ) Sertoli-Leydig cell tumor 5. A 67-year-old woman comes to the physician because of vaginal bleeding for 10 days. She has been soaking one sanitary pad daily. Menopause was 10 years ago. Her last Pap smear 5

years ago showed normal findings. Her temperature is 37 C (98.6 F), blood pressure is 128/78 mm Hg, pulse is 70/min, and respirations are 12/min. Pelvic examination shows a normal-appearing cervix and a small amount of blood at the cervical os. There is moderate thinning of the vaginal mucosa. The uterus is irregular and consistent in size with a 10-week gestation. There are no adnexal masses. Rectovaginal examination shows no abnormalities. Test of the stool for occult blood is negative. Which of the following is the most appropriate next step in diagnosis? A ) Transvaginal ultrasonography of the pelvis B ) Colposcopic-guided biopsy C ) Endometrial biopsy D ) Endometrial ablation E ) Culdocentesis 6. During a routine examination, a 32-year-old man has a blood pressure of 120/80 mm Hg. He is concerned because his father, grandfather, and two uncles have hypertension. He works as a systems programmer for a large computer company and frequently has to meet tight deadlines. He has smoked one pack of cigarettes daily for 10 years. He is 4.5 kg (10 lb) overweight and drinks three cups of coffee daily. Which of the following measures is most likely to reduce this patient's risk for hypertension over the next 5 years? A ) Increase intake of dietary fiber B ) Restrict caffeine C ) Stress management D ) Weight loss

7. A previously healthy 16-year-old boy comes to the physician because of persistent pain in his left testicle for 24 hours. He has not had any penile discharge. There is no history of trauma, but he plays soccer every day. Two months ago, he had sexual intercourse for the first time, and he used a condom. Examination shows an edematous, erythematous, exquisitely tender left scrotum that is lower than the right. Elevating the left testicle relieves the pain. The cremasteric reflex is present. Urinalysis shows 10 leukocytes/hpf and 1+ leukocyte esterase. A technetium 99m scan shows increased uptake in the left testicle. Which of the following is the most likely cause of the pain? A ) Cystitis B ) Epididymitis C ) Spermatocele D ) Testicular torsion E ) Testicular tumor 8. A 47-year-old woman comes to the physician because of persistent nonproductive cough for 6 weeks. She has not had fever or weight loss. She has hypertension treated with enalapril for the past 3 months. She does not smoke. There is no history of lung disease. She weighs 54 kg (120 lb) and is 163 cm (64 in) tall. Her temperature is 37 C (98.6 F), blood pressure is 130/80 mm Hg, pulse is 70/min, and respirations are 12/min. Examination and an x-ray film of the chest show no abnormalities. Which of the following is the most likely mechanism of this patient's cough? A ) Decreased plasma renin activity B ) Decreased serum angiotensin II levels C

) Increased serum angiotensin I levels D ) Increased serum bradykinin levels E ) Increased serum histamine levels 9. A 57-year-old man has been hospitalized for 2 days for treatment of unstable angina pectoris. He is currently receiving intravenous heparin and undergoing evaluation for coronary artery bypass grafting. His blood pressure is 160/90 mm Hg, pulse is 88/min, and respirations are 16/min. Laboratory studies show: Platelet count 90,000/mm3 Prothrombin time 12 sec (INR=1.1) Partial thromboplastin time 35 sec Which of the following is the most likely cause of these findings? A ) Excessive platelet destruction B ) Factor VIII deficiency C ) Inadequate platelet production D ) Uncontrolled activation of coagulation and fibrinolytic cascades E ) Vitamin K deficiency 10. A study is conducted to assess the effectiveness of a new blood test for early detection of prostate cancer. Ten thousand healthy men over the age of 50 years are randomly assigned to receive either annual rectal examination or annual screening with the new blood test. After 5 years, results show that of the 50 men in the blood test group that were diagnosed with prostate cancer, 40 were living 2 years after the diagnosis was made. In comparison, only 15 out of 45 men in the rectal examination group survived 2 years after being diagnosed with prostate cancer. Researchers conclude that the blood test increases survival compared with rectal examination. Which of the

following potential flaws is most likely to invalidate this conclusion? A ) Age of the patients B ) Diagnostic bias C ) Lead time bias D ) Recall bias E ) Type II error 11. A 62-year-old man comes to the emergency department because of progressive shortness of breath for 3 days. He has not had chest pain, orthopnea, or paroxysmal nocturnal dyspnea. He completed chemotherapy for small cell carcinoma of the lung 10 months ago. He has a history of twice nightly nocturia that has resolved over the past 3 days. He smoked two packs of cigarettes daily for 30 years but quit 1 year ago. His blood pressure is 96/60 mm Hg, and pulse is 116/min. There is jugular venous distention to the angle of the jaw. The lungs are clear to auscultation. Cardiac examination shows distant heart sounds, an S1 and S2, and no gallops or rubs. The liver has a span of 12 cm and is tender. There is no pedal edema. Laboratory studies show: Hemoglobin 10 g/dL Serum Na+ 135 mEq/L Cl 110 mEq/L K+ 4.2 mEq/L HCO3 22 mEq/L Urea nitrogen (BUN) 40 mg/dL Creatinine 1.6 mg/dL An ECG shows diminished amplitude of the QRS complexes. An x-ray film of the chest shows clear lung fields with an enlarged cardiac silhouette. Which of the following findings is most likely to be accentuated? A ) Cardiac output

B ) Fall in systolic arterial pressure with inspiration C ) Left ventricular end-diastolic pressure D ) Mitral regurgitation E ) Ventricular septal wall motion

12. A 35-year-old woman comes to the physician because of two 12-hour episodes of dizziness over the past 3 months. During episodes, she experiences the acute onset of rotatory vertigo and imbalance, decreased hearing, tinnitus, a sense of fullness of the right ear, and vomiting. Examination shows a mild hearing loss of the right ear. Which of the following is the most likely diagnosis? A ) Acoustic neuroma B ) Benign positional vertigo C ) Brain stem transient ischemic attacks D ) Meniere's disease E ) Viral labyrinthitis 3. An obese 33-year-old woman has had four 12-hour episodes of severe, sharp, penetrating pain in the right upper quadrant of the abdomen associated with vomiting but no fever. She has no diarrhea, dysuria, or jaundice and is asymptomatic between episodes. There is slight tenderness to deep palpation in the right upper quadrant. Which of the following is the most appropriate next step in diagnosis? A ) Supine and erect x-ray films of the abdomen B ) Upper gastrointestinal series

C ) Ultrasonography of the upper abdomen D ) CT scan of the abdomen E ) HIDA scan of the biliary tract 14. An otherwise healthy 19-year-old woman comes to the physician because of a 3-year history of intermittent facial blemishes. She drinks wine occasionally on weekends. She takes no medications. Examination shows multiple 1- to 2-mm red and white papules and larger red nodules on the forehead and cheeks. Which of the following is the most appropriate initial pharmacotherapy? A ) Oral isotretinoin B ) Systemic corticosteroids C ) Topical benzoyl peroxide D ) Topical corticosteroids E ) Topical metronidazole

15. A previously healthy 67-year-old man comes to the physician because of a 4-month history of hand weakness, intermittent tingling of the small fingers of his hands, and mild neck pain. Examination shows wasting, weakness, and fasciculations of the interossei muscles. Sensation is decreased to pinprick and vibration in the small fingers of each hand. Triceps tendon reflexes are decreased. Which of the following is the most likely diagnosis? A ) Amyotrophic lateral sclerosis B ) Cervical spondylosis C ) Multiple sclerosis

D ) Myasthenia gravis E ) Myasthenic (Lambert-Eaton) syndrome F ) Myotonic muscular dystrophy G ) Polymyalgia rheumatica H ) Polymyositis I ) Progressive neuropathic (peroneal) muscular atrophY

16. A 19-year-old college student comes to student health services because of constant worrying since starting his freshman year 6 months ago. He attends college several hundred miles away from his hometown. He reports feeling constantly scrutinized by other students and professors. He feels embarrassed and anxious in class and is in constant fear of blushing. He has started to skip his classes because of his anxiety and worries that he will not be able to complete the school year. He drinks alcohol occasionally because it helps him overcome his fear of being embarrassed in front of others. He does not use illicit drugs. Physical examination shows no abnormalities. On mental status examination, he is mildly anxious. He exhibits fair eye contact and shifts uncomfortably in his chair. Laboratory studies are within normal limits. Which of the following is the most appropriate pharmacotherapy for this patient? A ) Disulfiram B ) Donepezil C ) Fluoxetine D ) Haloperidol

E ) Methylphenidate F ) Valproic acid 17. A 42-year-old woman comes for a follow-up examination. Two weeks ago, her blood pressure was 152/94 mm Hg during a routine visit. Her blood pressure today is 150/94 mm Hg, pulse is 76/min, and respirations are 14/min. Examination shows no other abnormalities. Serum studies show: Na+ 142 mEq/L Cl 105 mEq/L K+ 4 mEq/L HCO3 26 mEq/L Urea nitrogen (BUN) 12 mg/dL Glucose 101 mg/dL Creatinine 0.8 mg/dL An ECG shows no abnormalities. Which of the following is the most appropriate next step in management? A ) Measurement of plasma renin activity B ) Serum lipid studies C ) 24-Hour urine collection for measurement of metanephrine level D ) Echocardiography E ) Captopril renal scan 18. A 6-year-old boy with cystic fibrosis is brought to the physician by his mother because his skin has been cool and clammy for 30 minutes. Earlier in the day, he had been playing outdoors, and the temperature was 99 F. When returning indoors, he was thirsty and restless. His blood pressure is 70/40 mm Hg, and pulse is 120/min. Examination shows dry mucous membranes. Serum sodium level is 128 mEq/L, and serum chloride level is 87 mEq/L. Which of the following is the most likely

explanation for these findings? A ) Excessive sweat electrolyte level B ) Excessive sweat volume C ) Excessive urinary output D ) Excessive vasopressor secretion E ) Inadequate sweat production

19. A 27-year-old primigravid woman at 29 weeks' gestation comes to the emergency department because of a 24-hour history of increasingly severe right-sided abdominal pain and no appetite. She has vomited twice over the past 4 hours. She has not had vaginal bleeding. Her temperature is 38.2 C (100.8 F). Examination shows exquisite tenderness of the right lateral flank and the fundus. There are no peritoneal signs. Bowel sounds are absent. The fetal heart rate is 144/min. Laboratory studies show: Hematocrit 37% Leukocyte count 16,000/mm3 Serum Total bilirubin 1.1 mg/dL Amylase 32 U/L Lactate dehydrogenase 110 U/L Urine WBC 35/hpf Which of the following is the most likely diagnosis? A ) Abruptio placentae B ) Appendicitis C ) Cholelithiasis D ) Colitis E ) Pyelonephritis

20. A 32-year-old woman has had a lump in her neck for 6 months. She has a 1.2-cm solitary left thyroid nodule. Fine-needle aspiration cytology of the mass is consistent with a low-grade papillary malignancy. Which of the following is the most appropriate next step in management? A ) Thyroid scan B ) 131I therapy C ) Propylthiouracil therapy D ) Thyroxine therapy E ) Left thyroid lobectomy For each patient with vaginal bleeding, select the most likely diagnosis. A ) Bacterial vaginosis B ) Precocious puberty C ) Urinary tract infection D ) Vaginal foreign body E ) Vaginal laceration F ) von Willebrand's disease 21. A 12-year-old girl is brought to the physician by her mother because of heavy vaginal bleeding since her first menstrual period began 1 week ago. She has to change sanitary pads every 2 hours, and her mother is concerned that this is not normal. Her blood pressure is 80/60 mm Hg, and pulse is 110/min. Breast and axillary and pubic hair development are Tanner stage 4. Pelvic examination shows normal external

genitalia, a small normal-appearing cervix, and a small uterus. Her hemoglobin level is 7 g/dL.

22. A 7-year-old girl is brought to the physician by her mother 4 hours after she noticed that her daughter had blood on her underwear. She has no signs of sexual development. Examination shows normal external genitalia. There is a foul-smelling discharge. 23. A 10-year-old girl is brought to the physician because of temperatures to 40 C (104 F) and headaches for 1 week. Her mother says that fever and chills occur every other day and typically last for several hours. She appears ill and is lethargic during febrile episodes. She and her family returned from a trip to West Africa 2 weeks ago. She currently appears ill. Her temperature is 40 C (104 F), blood pressure is 94/64 mm Hg, pulse is 146/min, and respirations are 20/min. Examination shows mild scleral icterus and conjunctival pallor. The liver edge is palpated 3 cm below the right costal margin, and the spleen tip is palpated 3 cm below the left costal margin. Laboratory studies show: Hemoglobin 8 g/dL Leukocyte count 6400/mm3 Segmented neutrophils 46% Eosinophils 5% Lymphocytes 40% Monocytes 9% Serum Na+ 132 mEq/L Cl 98 mEq/L K+ 4.2 mEq/L HCO3 16 mEq/L Urea nitrogen (BUN) 21 mg/dL Bilirubin Total 5.2 mg/dL Direct 0.8 mg/dL Aspartate aminotransferase (AST, GOT) 64 U/L Alanine aminotransferase (ALT, GPT) 98 U/L Urine Color tea Blood strongly positive RBC occasional

Which of the following measures is most likely to have prevented this condition? A ) Hepatitis A vaccine B ) Typhoid vaccine C ) Oral isoniazid prophylaxis D ) Oral mefloquine prophylaxis E ) Oral trimethoprim-sulfamethoxazole prophylaxis F ) Intramuscular immune globulin

24. A 6-month-old boy is brought to the physician because of seizures over the past week. His parents note that during this period, he has had 10 to 20 episodes of throwing out his arms for 1 to 2 seconds then crying as if he were afraid of something. He was born at term following an uncomplicated pregnancy and delivery. Development is appropriate for age. Two days ago, treatment with amoxicillin was begun for otitis media. He appears pale and apathetic. Examination shows multiple white patches over the skin that become more distinct with Wood's lamp examination. Neurologic examination shows no abnormalities. An MRI of the brain is shown. Which of the following is the most likely diagnosis? A ) Congenital cytomegalovirus encephalopathy B ) Congenital toxoplasmosis encephalopathy C ) Hypoxic-ischemic encephalopathy D ) Neurofibromatosis E ) Tuberous sclerosis 25.

An ELISA is used to detect Chlamydia trachomatis infection in patients seen at a family planning clinic. In the first 500 patients, cultures are also done to check the accuracy of the ELISA. The results are listed below: Chlamydia culture + ELISA + 38 5 2 455 Which of the following is the positive predictive value of the ELISA? A ) 2/40 B ) 5/43 C ) 38/40 D ) 38/43 E ) 38/493

26. A 27-year-old woman is brought to the emergency department by her mother who found her comatose 30 minutes ago. Her mother says that her daughter had been having lower abdominal pain and vaginal bleeding over the past week. The patient had an ectopic pregnancy 2 years ago and was also treated with doxycycline for pelvic inflammatory disease at that time. Her blood pressure is 40/20 mm Hg, pulse is 160/min, and respirations are 24/min. The abdomen is distended and rigid with

decreased bowel sounds. Hemoglobin level is 4.2 g/dL, and leukocyte count is 12,500/mm3. Culdocentesis is positive. Which of the following is the most appropriate next step in management? A ) Bromocriptine therapy B ) Clomiphene therapy C ) Conjugated estrogen therapy D ) Ergot derivative therapy E ) Hysteroscopy F ) Laparoscopy G ) Dilatation and curettage H ) Endometrial ablation I ) Exploratory laparotomy J ) Total abdominal hysterectomy

27. A 43-year-old man comes to the emergency department because of fever, chills, malaise, cough, and pleuritic chest pain for 3 days; the cough is productive of foul-smelling, purulent sputum. He has alcoholism. His temperature is 39.2 C (102.6 F), blood pressure is 110/70 mm Hg, pulse is 120/min, and respirations are 14/min. Hemoglobin level is 15 g/dL, and leukocyte count is 25,000/mm3. An ECG shows normal findings. An x-ray film of the chest shows a 4-cm cavity in the superior segment of the right lower lobe and an air-fluid level. While awaiting results of sputum culture, which of the following is the most appropriate next step in management? A ) Observation B

) Broad-spectrum antibiotic therapy C ) Isoniazid therapy D ) Intravenous amphotericin B therapy E ) Tube thoracostomy 28. A 12-year-old girl with type 1 diabetes mellitus is brought to the physician because of shortness of breath and fatigue for 1 day. Since menarche began 4 months ago, she has had one episode of diabetic ketoacidosis per month; prior to that she had been stable. Her blood pressure is 110/70 mm Hg, pulse is 140/min, and respirations are 36/min. She appears to be moderately dehydrated. Laboratory studies show: Serum Na+ 132 mEq/L Cl 90 mEq/L K+ 5.9 mEq/L HCO3 6 mEq/L Urea nitrogen (BUN) 48 mg/dL Glucose 600 mg/dL Creatinine 2.8 mg/dL Urine Glucose 4+ Ketones 3+ Protein 1+ Which of the following is the most likely underlying cause of this patient's respiratory disorder? A ) Acute interstitial nephritis B ) Acute renal failure C ) Diabetic nephropathy D ) Hyperinsulinemia E ) Increased plasma ketone level 29. A 28-year-old woman develops fatigue and

orthopnea 3 weeks after the uncomplicated delivery of her first child. Her blood pressure is 115/78 mm Hg, and pulse is 112/min. Crackles are heard at both lung bases. The point of maximal impulse is laterally displaced and diffuse; an S3 is present. There is 2+ pedal edema. An x-ray film of the chest shows vascular cephalization of pulmonary vasculature and Kerley B lines. Which of the following is the most appropriate next step in management? A ) Measurement of serum creatine kinase and lactate dehydrogenase activities B ) Echocardiography C ) Dipyridamole thallium scan D ) Ventilation-perfusion lung scans E ) Myocardial biopsy 30. A previously healthy 22-year-old woman comes to the physician because of a paroxysmal cough for 2 weeks. She works as an oncology nurse. Her temperature is 37 C (98.6 F). Examination shows no abnormalities except for intermittent coughing spells during the examination. An x-ray film of the chest is normal. Cultures of nasopharyngeal secretions grow Bordetella pertussis. Which of the following is the most appropriate pharmacotherapy for this patient? A ) Amoxicillin B ) Amoxicillin-clavulanate C ) Cefprozil D ) Erythromycin E ) Penicillin G

31. A 5-year-old girl is brought to the physician because of a 2-day history of temperatures to 39.5 C (103.1 F) and pain in the right side. She has had two episodes of vomiting during this period but no diarrhea or symptoms of upper respiratory tract infection. She has vesicoureteral reflux and a history of recurrent urinary tract infections. She appears ill. Her temperature is 39.8 C (103.6 F), blood pressure is 110/60 mm Hg, pulse is 150/min, and respirations are 25/min. Examination shows right-sided costovertebral angle tenderness. In addition to obtaining urine cultures, which of the following is the most appropriate next step in management? A ) Intravenous pyelography B ) Renal ultrasonography C ) Voiding cystourethrography D ) Intramuscular antibiotic therapy and reexamination in 24 hours E ) Intravenous antibiotic therapy 32. A 10-year-old boy is brought to the physician because of weakness and decreased appetite for 3 months. He weighs 30 kg (66 lb) and is 142 cm (56 in) tall; he has had a 2.3-kg (5-lb) weight loss since his last examination 6 months ago. He appears thin. Deep tendon reflexes are brisk. Chvostek's sign is positive. His serum calcium level is 6.5 mg/dL, and serum intact parathyroid hormone level is 190 pg/mL (N=965). In addition to calcium supplementation, which of the following is the most appropriate next step in management? A ) Oral phosphate supplementation B ) Oral vitamin D supplementation C ) Intranasal calcitonin therapy

D ) Parenteral bisphosphate therapy E ) Parenteral parathyroid hormone therapy 33. A healthy 8-year-old girl is brought to the physician in July for a well-child examination. Her mother says that her daughter is spending the summer at a nearby lake. Over the past month, she has had two episodes of painful sunburn despite her mother's efforts, including SPF 25 sunblock just before she goes swimming and urging her to wear a hat and long-sleeved garments. The child takes no medications. She has blond hair, blue eyes, and a fair complexion. The mother seeks advice about preventing further sun damage to her child's skin. Which of the following is the most appropriate recommendation? A ) Prohibit swimming on cloudless days B ) Apply the sunblock lotion 45 minutes before swimming C ) Change to a higher-level SPF lotion D ) Apply Burrow's solution compresses after each overexposure E ) Daily use of antioxidant vitamin supplement F ) Early treatment of any sun overexposure with topical corticosteroids

34. Two days after admission to the hospital because of a 3-day history of slurred speech, double vision, and dysphagia, a 24-year-old woman becomes quadriplegic and requires intubation and mechanical ventilation. Her medical history is unremarkable. One week ago, she attended a family picnic; several of her family members have had abdominal cramps and diarrhea since the picnic. Her temperature is 37

C (98.6 F), blood pressure is 120/80 mm Hg, and pulse is 120/min. Examination shows dry mucous membranes, large unreactive pupils, ophthalmoplegia, and profound facial weakness. There is areflexia, quadriplegia, and no movement of the palate and tongue. Sensation is normal. Babinski's sign is absent. Which of the following is the most appropriate pharmacotherapy? A ) Antitoxin B ) Azathioprine C ) Interferon D ) Pyridostigmine E ) Riluzole

35. A 2325-g (5 lb 2 oz) male newborn is delivered at 33 weeks' gestation; Apgar scores are 7 and 8 at 1 and 5 minutes, respectively. The 13-year-old mother had no prenatal care and did not know how much weight she gained. During the pregnancy, the mother smoked marijuana and took over-the-counter vitamins occasionally; she did not drink alcohol and had no illness except for an upper respiratory tract infection 4 months ago. She did not know she was pregnant until 2 weeks ago; her family is unaware of her condition. She has had one sexual partner. During the hospital stay, the newborn and his mother have no complications. The newborn is at greatest risk for morbidity and mortality from which of the following? A ) Child abuse B ) Congenital syphilis C ) Hypocalcemia D ) Lead poisoning

E ) Seizures 36. A 2-month-old girl is brought to the physician because of a 2-week history of progressive difficulty breathing and poor feeding. She has had rapid and labored breathing and sweating during feedings. There is no history of fever or viral illness. Her temperature is 37 C (98.6 F), blood pressure is 80/60 mm Hg, pulse is 130/min, and respirations are 40/min. Bilateral crackles are heard at both lung bases. A grade 4/6 holosystolic murmur is heard along the left sternal border; the precordium is hyperdynamic. The liver edge is palpated 4 cm below the right costal margin. An x-ray film of the chest shows cardiomegaly and pulmonary congestion. Which of the following is the most likely underlying mechanism for this child's condition? A ) Decreased systemic vascular resistance B ) Increased pulmonary vascular resistance C ) Increased systemic vascular resistance D ) Intracardiac left-to-right shunt E ) Intracardiac right-to-left shunt

37. A 37-year-old man comes to the physician 6 months prior to traveling to sub-Saharan Africa for 1 year. He has no history of hepatitis and has no high-risk behavior for hepatitis B. Which of the following is the most appropriate recommendation to prevent hepatitis during and after his trip? A ) Immune globulin B ) Hepatitis B immune globulin (HBIG) only C ) Hepatitis B vaccine series only

D ) Hepatitis B vaccine series and HBIG E ) Hepatitis B vaccine series and hepatitis A vaccine

38. A 67-year-old man comes to the physician because of a 2-month history of progressive shortness of breath. He has had a 4.5-kg (10-lb) weight loss over the past 4 months. He has not had chest pain. He has congestive heart failure treated with furosemide, digoxin, and enalapril. He has smoked two packs of cigarettes daily for 30 years. He appears alert and is in no acute distress. His temperature is 37.2 C (99 F), blood pressure is 140/85 mm Hg, pulse is 84/min, and respirations are 18/min. Examination shows no jugular venous distention. There is dullness to percussion, and breath sounds are decreased at the left base. Cardiac examination shows a laterally displaced point of maximal impulse, normal S1 and S2, and an S3 at the apex. There is 1+ edema over the extremities. An x-ray film of the chest shows an enlarged cardiac silhouette, left hilar fullness, and a moderate-sized left pleural effusion. Thoracentesis yields straw-colored fluid. Laboratory studies show: Serum Glucose 90 mg/dL Protein 7 g/dL Lactate dehydrogenase 300 U/L Pleural fluid pH 7.25 Glucose 75 mg/dL Protein 4.5 g/dL Lactate dehydrogenase 280 U/L Leukocyte count 2000/mm3 Segmented neutrophils 15% Lymphocytes 85% A Gram's stain and acid-fast stains are negative for any organisms. Which of the following is the most likely cause of this patient's pleural effusion? A

) Bacterial pneumonia B ) Collagen vascular disease C ) Congestive heart failure D ) Malignancy E ) Pulmonary embolus with infarction F ) Viral pleuritis 39. A previously healthy 67-year-old woman is admitted to the hospital because of a 2-week history of dark urine, clay-colored stools, and increasing jaundice. She has had a 9-kg (20-lb) weight loss over the past 2 months due to loss of appetite. She also has had generalized itching that is most severe at night. She has not had any abdominal pain. Examination shows no abnormalities except for jaundice. Which of the following is the most likely diagnosis? A ) Common bile duct stone B ) Drug-induced jaundice C ) Hemolytic jaundice D ) Pancreatic carcinoma E ) Viral hepatitis 40. A 2-year-old boy is brought to the physician because of fever and listlessness for 12 hours. He has had recurrent episodes of pneumonia and otitis media over the past year. Two maternal uncles died of pneumonia in early childhood. One year ago, he was at the 50th percentile for height and weight; he is currently at the 25th percentile for height and 10th percentile for weight. He appears ill. His temperature is 39 C (102.2 F), blood pressure is 60/40 mm Hg, pulse is 160/min, and

respirations are 36/min. Examination shows cool and mottled extremities. A blood culture grows Streptococcus pneumoniae. Serum IgE, IgG, and IgM levels are markedly decreased. Which of the following diagnostic tests is most likely to be abnormal? A ) Candidal skin test B ) Flow cytometry identification of B lymphocytes C ) Nitroblue tetrazolium test D ) T-lymphocyte receptor stimulation by concanavalin A E ) Total serum hemolytic complement assay

41. A 1-week-old newborn has had poor feeding, vomiting, and progressive lethargy over the past 4 days. She was born at term; pregnancy, labor, and delivery were uncomplicated, and she had no congenital anomalies. She is being breast-fed. She has a healthy 2-year-old brother; a sister died at 10 days of age after a full-term birth. Examination shows decreased muscle tone and poor responsiveness; reflexes are normal. Serum bicarbonate level is 8 mEq/L, pH is 7.15, and plasma ammonia level is 10 times the upper limit of normal. Which of the following is the most likely cause? A ) Mitochondrial disorder B ) Mucopolysaccharidoses disorder C ) Organic acid metabolism disorder D ) Renal tubular acidosis E ) X-linked leukodystrophy

For each patient with an infection, select the most likely causal organism. A ) Chlamydia trachomatis B ) Clostridium botulinum C ) Clostridium tetani D ) Group A streptococcus E ) Group B streptococcus F ) Lister ia monocytogenes G ) Neisseria gonorrhoeae H ) Streptococcus pneumoniae 42. A premature 18-hour-old newborn is intubated and mechanically ventilated because of progressive respiratory distress. Labor was complicated by maternal fever and increased leukocyte count; membranes ruptured 36 hours before delivery. His blood pressure and urine output have decreased since birth. The newborn appears acutely ill, and peripheral pulses are not palpable; the skin is pale, cool, and mottled. The liver edge is palpable 4 cm below the right costal margin. His leukocyte count is 5000/mm3, platelet count is 48,000/mm3, and partial thromboplastin time is 60 sec. An x-ray film of the chest shows diffuse, bilateral, interstitial infiltrates.

43. A 6-year-old boy has been unable to walk for 2 days because of a sore right knee. Three weeks ago he had a sore throat and fever that resolved within 2 days. He appears acutely ill. His temperature is 39.2 C (102.5 F), and pulse is 120/min. A grade 2/6 pansystolic murmur is

heard at the apex. The right knee is red, tender, and swollen; any motion is painful. His leukocyte count is 15,000/mm3, and erythrocyte sedimentation rate is 120 mm

44. A 70-year-old woman comes to the physician because of fatigue and increasing difficulty in her daily functioning over the past 2 months. During the day, she lies in bed for hours and cries. She has had a 10.9-kg (24-lb) weight loss over the past 4 weeks, eats only cookies, has lost interest in almost everything, and wishes to kill herself. Her husband died 4 months ago. Physical examination and laboratory studies show normal findings except for decreased serum albumin and total protein levels. Which of the following is the most likely diagnosis? A ) Adjustment disorder with depressed mood B ) Bereavement C ) Bipolar disorder, depressed D ) Dysthymic disorder E ) Major depressive disorder

45. A 42-year-old woman comes to the physician because of increasing low back pain for 2 days. She is a daily intravenous drug user. She has a history of pyelonephritis, abscesses at injection sites, and pelvic inflammatory disease. Her temperature is 39 C (102.2 F), blood pressure is 130/70 mm Hg, pulse is 84/min, and respirations are 20/min. Examination shows warm, dry skin, a supple neck, and no jugular venous distention. There is tenderness over L4. Pelvic examination shows mild erythema around the cervical os and scant discharge; there is no adnexal or cervical motion tenderness. Her hematocrit is 30%,

leukocyte count is 10,600/mm3, and serum glucose level is 110 mg/dL. Urinalysis is within normal limits. Which of the following is the most appropriate next step in management? A ) Echocardiography B ) Renal ultrasonography C ) MRI of the back D ) Colposcopy E ) Laparoscopy 46. A 13-year-old boy is brought to the physician by his mother because of frequent headaches over the past 5 weeks. He describes the headaches as dull pain across his forehead; they occur four to five times weekly. Acetaminophen does not relieve the pain. His mother states that occasionally he has nausea without vomiting. He has schizophrenia well controlled with risperidone. Physical examination shows no abnormalities. An MRI of the brain is most likely to show which of the following? A ) Bilateral increased caudate nuclei B ) Decreased cerebellar volume C ) Hippocampal symmetry D ) Increased lateral ventricle size E ) Multiple white-matter hyperintensities Report Abuse * block 3:--dolly123 - 11/07/06 17:59 #548031

1. A 42-year-old woman, gravida 2, para 2, comes to

the physician because of a 3-month history of swelling of her legs and mild abdominal pain and bloating. Abdominal examination shows no abnormalities. Rectovaginal examination shows fullness in the right adnexa. Transvaginal ultrasonography shows an irregular mass in the right ovary with some solid components to a predominantly cystic lesion. Her serum CA 125 level is 120 U/mL (N<35). Treatment with which of the following is most likely to have prevented this patient's symptoms? A ) Antiestrogens B ) Antiprogestationals C ) Medroxyprogesterone D ) Oral contraceptives E ) Ovulation-inducing drugs

2. A 15-year-old girl is brought to the physician 3 months after she had a blood pressure of 150/95 mm Hg at a routine examination prior to participation in school sports. She is asymptomatic and has no history of serious illness. Twelve months ago, she was diagnosed with a urinary tract infection and treated with oral trimethoprim-sulfamethoxazole. She currently takes no medications. Subsequent blood pressure measurements on three separate occasions since the last visit have been: 155/94 mm Hg, 145/90 mm Hg, and 150/92 mm Hg. She is at the 50th percentile for height and 95th percentile for weight. Her blood pressure today is 150/90 mm Hg confirmed by a second measurement, pulse is 80/min, and respirations are 12/min. Examination shows no other abnormalities. Her hematocrit is 40%. Urinalysis is within normal limits. Cardiac and renal ultrasonography shows no abnormalities. Which of the following is the most appropriate next step in management? A

) Exercise and weight reduction program B ) Measurement of urine catecholamine levels C ) Measurement of urine corticosteroid levels D ) Captopril therapy E ) Hydrochlorothiazide therapy

3. Over the past 4 years, a 40-year-old woman has had increasing episodes of loss of urine and difficulty emptying her bladder. She has had no dysuria. She has a 30-year history of type 1 diabetes mellitus. She weighs 66 kg (145 lb) and is 175 cm (69 in) tall. Pelvic examination shows a moderate cystocele. Postvoiding catheterization yields 700 mL of clear urine. Which of the following is the most likely cause of the patient's genitourinary symptoms? A ) Carcinoma of the bladder B ) Detrusor instability C ) Neurogenic bladder D ) Urethral diverticulum E ) Uterine prolapse

4. A 57-year-old man comes to the emergency department because of an episode of confusion, nervousness, sweating, and palpitations 1 hour ago. He has had four similar episodes over the past 3 weeks; they last 2 to 3 minutes and are relieved by ingesting solid food or liquids. His blood pressure is 140/74 mm Hg, pulse is 76/min, and respirations are 18/min. Examination shows no other abnormalities. The most appropriate next step in diagnosis is serum measurement of which of the following?

A ) Fasting gastrin level B ) Fasting insulin and glucose levels C ) Glucagon level D ) Glucose and somatostatin level E ) Glucose and vasoactive intestinal 5. An otherwise healthy 15-year-old girl is brought to the physician because she has never had a menstrual period. She reports that breast development started 1 year ago and pubic and axillary hair development began 6 months ago. Examination shows normal genitalia. Breast development is Tanner stage 4, and pubic hair development is Tanner stage 3. Which of the following is the most appropriate next step in management? A ) Reexamination in 1 year if the patient has not had menarche B ) Measurement of serum follicle-stimulating hormone and luteinizing hormone levels C ) Measurement of serum thyroid-stimulating hormone and prolactin levels D ) Karyotype analysis E ) Progesterone withdrawal test F ) Pelvic ultrasonographypolypeptide levels

6. A 2-month-old boy is brought to the physician because of a 6-week history of persistent diarrhea and vomiting, most pronounced after formula feedings. He has had a 113-g (4-oz) weight loss since

birth. He currently weighs 3100 g (6 lb 13 oz) and is 51 cm (20 in) in length. He appears irritable. Examination shows jaundice. The lungs are clear to auscultation. No murmurs are heard. The liver is palpated 2 to 3 cm below the right costal margin, and the spleen is palpated 1 to 2 cm below the left costal margin. Laboratory studies show: Serum Glucose 35 mg/dL Bilirubin (total) 2.3 mg/dL Urine Glucose negative Reducing substances 3+ Which of the following is the most likely mechanism of these findings? A ) Decreased gluconeogenesis B ) Decreased insulin secretion C ) Increased glucagon secretion D ) Increased gluconeogenesis E ) Increased insulin secretion F ) Insulin resistance

7. A 75-year-old man has had hypertension for 25 years. There is an unusually prominent pulsation of the abdominal aorta in the upper midabdomen. A systolic bruit is heard at this site. Femoral, popliteal, and pedal pulses are present. Which of the following is the most appropriate initial diagnostic study? A ) X-ray film of the abdomen B ) Abdominal ultrasonography C ) Doppler ultrasonography of the arteries of the legs

D ) Abdominal aortography E ) Intravenous pyelography

8. A 14-year-old boy is brought to the physician because of constant right knee pain for 2 weeks. The pain is not relieved by rest or analgesics. There is no history of trauma. He is at the 50th percentile for height and above the 95th percentile for weight. He walks with a limp but is not in distress. His vital signs are within normal limits. Examination shows full range of motion of both knees; passive abduction and internal rotation of the right hip produce pain. There is no swelling of either knee or hip. Neurologic examination shows no abnormalities. Which of the following is the most likely diagnosis? A ) Juvenile rheumatoid arthritis B ) Osgood-Schlatter disease C ) Septic arthritis D ) Slipped capital femoral epiphysis E ) Toxic synovitis

9. A previously healthy 72-year-old man comes to the physician because of a 2-year history of hand tremors and progressive difficulty walking. He lives alone and has no close relatives. He is alert and oriented. Physical examination shows a decreased rate of eye blinking. Neurologic examination shows masked facies and a pill-rolling resting tremor of both hands. There is cogwheel rigidity of the upper extremities and generalized bradykinesia. His handwriting has become small and illegible. He has a slow, shuffling, festinating gait with a tendency to lean forward. Postural reflexes are impaired. This patient is at

greatest risk for injury due to which of the following? A ) Bradykinesia B ) Cogwheel rigidity C ) Decreased rate of eye blinking D ) Postural reflex impairment E ) Tremor

10. A 47-year-old woman comes for a routine health maintenance examination. She has a 10-year history of type 2 diabetes mellitus that is well controlled with an oral hypoglycemic agent. Her mother died of a myocardial infarction at the age of 38 years. The patient weighs 82 kg (180 lb) and is 163 cm (64 in) tall. Her blood pressure is 150/95 mm Hg. Examination shows multiple small skin tags below the eyebrows and on the nose and eyelids. Which of the following is the most appropriate next step in diagnosis? A ) Measurement of serum follicle-stimulating hormone level B ) Serum lipid studies C ) 24-Hour urine collection for measurement of creatinine clearance D ) X-ray film of the chest E ) ECG F ) No further studies indicated

11.

A previously healthy 15-year-old boy is brought to the physician because of a 5-day history of fever, intractable nausea and vomiting, sore throat, and muscle pain. His mother has been giving him ibuprofen and amoxicillin that was remaining from a previous streptococcal throat infection. He appears ill, and his lips are parched. His temperature is 38.9 C (102 F), blood pressure is 120/74 mm Hg while supine and 100/70 mm Hg while standing, and pulse is 92/min while supine and 120/min while standing. Examination shows dry mucous membranes. The oropharynx is erythematous without exudate. There is shotty cervical adenopathy. The abdomen is soft without organomegaly. Laboratory studies show: Serum Na+ 138 mEq/L Cl 98 mEq/L K+ 3.4 mEq/L HCO3 21 mEq/L Urea nitrogen (BUN) 55 mg/dL Glucose 105 mg/dL Creatinine 1.3 mg/dL Amylase 40 U/L Urine Ketones moderate WBC negative RBC negative Na+ 8 mEq/L Protein negative Which of the following is the most likely explanation for this patient's renal insufficiency? A ) Acute tubular necrosis B ) Amoxicillin-induced acute interstitial nephritis C ) Ibuprofen-induced renal failure D ) Post-streptococcal glomerulonephritis E ) Severe volume depletion

12. A 77-year-old woman is brought to the emergency department after collapsing at home. Six hours ago, she had the sudden

onset of massive bright red rectal bleeding. On arrival, her blood pressure is 90/60 mm Hg, and pulse is 120/min. Abdominal examination shows no abnormalities. Insertion of a nasogastric tube yields clear aspirate. Her hematocrit is 28%. Which of the following is the most likely diagnosis? A ) Colon cancer B ) Diverticulosis C ) Duodenal ulcer D ) Hemorrhoids E ) Inflammatory bowel disease

13. A previously healthy 27-year-old woman comes to the emergency department 1 hour after a 30-minute episode of shortness of breath, nausea, diaphoresis, and crushing substernal chest pain radiating to the left shoulder; the episode resolved spontaneously. She is currently asymptomatic. Her blood pressure is 110/84 mm Hg, pulse is 70/min, and respirations are 16/min. Examination shows no abnormalities. An ECG shows a normal sinus rhythm and no other abnormalities. Ten minutes later, she sits up in bed stating that her symptoms have returned. Her blood pressure now is 150/105 mm Hg, pulse is 120/min, and respirations are 24/min. A repeat ECG shows 5-mm ST-segment elevation in leads II, III, and aVF. Her symptoms resolve after administration of aspirin and nitroglycerin. An angiography is ordered and shows no evidence of coronary atherosclerosis. Which of the following is the most appropriate pharmacotherapy? A ) Angiotensin-converting enzyme (ACE) inhibitor B ) Aspirin C

) Benzodiazepine D ) Calcium-channel blocking agent E ) Corticosteroids F ) Thiazide diuretic

Na+(mEq/L) Cl(mEq/L) K+(mEq/L) HCO3(mEq/L) pH Specific gravity A ) 132 89 2.8 39 5.0 1.025 B ) 133 110 3.9 16 6.0 1.015 C ) 163 117 4.3 22 5.5 1.003 D ) 165 115 4.5 19 5.0 1.030 14. An 8-week-old boy is brought to the physician by his mother because of persistent vomiting for 2 days. He has spit up intermittently since birth. He has had no diarrhea. He appears irritable. His temperature is 38 C (100.4 F), blood pressure is 90/60 mm Hg, pulse is 130/min, and respirations are 25/min. Examination shows a sunken anterior fontanelle. Mucous membranes are dry. The lungs are clear to auscultation. Heart sounds are normal. The abdomen is scaphoid, and bowel sounds are active.

15. A 7-week-old boy is brought to the physician because of fever, irritability, and lethargy for 3 days. He has had no vomiting, diarrhea, or symptoms of an upper respiratory tract infection. He always seems hungry to his mother despite being fed 3 ounces of formula every 3 hours. His mother also notes that he has many wet diapers throughout the

day and night. He currently appears irritable. His temperature is 37.8 C (100 F), blood pressure is 80/50 mm Hg, pulse is 150/min, and respirations are 25/min. Examination shows a sunken anterior fontanelle. Mucous membranes are dry. There is tenting of the skin. Examination shows no other abnormalities. Urinalysis is negative for glucose and protein.

16. A 57-year-old woman is extubated and transferred to the recovery room after a cholecystectomy. She appears restless. Her blood pressure is 120/70 mm Hg, pulse is 80/min, and respirations are 10/min. Arterial blood gas analysis on room air shows: pH 7.24 PCO2 85 mm Hg PO2 60 mm Hg Intravenous naloxone therapy is begun, but she does not improve. Which of the following is the most appropriate next step in management? A ) Encouraging deep breathing and cough B ) Administration of 40% oxygen via nasal cannula C ) Administration of furosemide D ) Transfusion of 1 unit of packed red blood cells E ) Reintubation and mechanical ventilation

17. A previously healthy 24-year-old woman is brought to the physician by her husband because of several episodes of loss of consciousness over the past 4 days. Her husband reports that during episodes, she jerks her arms and legs wildly. Each episode lasts up to 1 hour; between episodes, her behavior is normal. She is planning to move to another state because of her husband's work. She has been

extremely anxious and upset about the move because she will have to leave her mother, who was recently diagnosed with breast cancer. There is no family history of seizure disorder. Her temperature is 36.7 C (98 F), blood pressure is 130/80 mm Hg, pulse is 84/min, and respirations are 18/min. Neurologic examination shows no abnormalities. Electroencephalography shows normal findings during an episode of shaking. Which of the following is the most likely underlying cause? A ) Catatonia B ) Complex partial seizure C ) Conversion reaction D ) Dissociative fugue E ) Malingering F ) Tonic-clonic seizure

18. A 37-year-old man comes to the physician because of a 3-month history of increasing pain of his shoulders and upper arms. Over the past 2 weeks, he has also had difficulty lifting heavy objects off shelves in his garage. Two days ago, he burned his hand by touching a hot pan and felt little discomfort. He sustained a concussion in a motor vehicle collision at the age of 29 years. He has no allergies. He takes no medications. He does not smoke cigarettes or use illicit drugs, and he drinks one to two beers daily. Muscle strength is decreased equally in both arms. Temperature and pain sensation are decreased in both hands, but light touch is normal. Muscle strength and sensation are normal in the lower extremities. Neurologic examination shows no other abnormalities. Which of the following is the most likely diagnosis? A ) Alcoholic peripheral neuropathy

B ) Ankylosing spondylitis C ) Guillain-Barr syndrome D ) Herniated intervertebral disc E ) Multiple sclerosis F ) Polymyositis G ) Syringomyelia

19. A 62-year-old woman with ovarian cancer comes to the emergency department because of fever for 2 days. Ten days ago, she received chemotherapy with paclitaxel and carboplatin. She feels tired but has not had nausea or vomiting. Her temperature is 39.5 C (103.1 F), blood pressure is 100/60 mm Hg, and pulse is 115/min. The lungs are clear to auscultation. Examination shows a soft, nontender abdomen. Her hematocrit is 32%, leukocyte count is 800/mm3, and platelet count is 105,000/mm3. Serum electrolyte levels are within normal limits. Which of the following is the most appropriate next step in treatment? A ) Plasmapheresis B ) Additional chemotherapy C ) Intravenous antibiotic therapy D ) Intravenous corticosteroid therapy E ) Transfusion of 2 units of leukocytes F ) Transfusion of 2 units of packed red blood cells

20.

A 47-year-old woman who is visiting from Australia comes to the physician because of increasing urine output over the past month. She has had no dysuria or hematuria. She has a history of chronic headaches, peptic ulcer disease, and urinary tract infections. An evaluation 18 months ago for headaches, including CT scan of the head, showed no abnormalities; treatment with ibuprofen and phenacetin was initiated at that time, and her headaches have been well controlled. Her temperature is 37.1 C (98.8 F), blood pressure is 140/82 mm Hg, pulse is 78/min, and respirations are 14/min. Examination shows no abnormalities. Laboratory studies show: Hematocrit 32% Mean corpuscular volume 88 m3 Serum Glucose 130 mg/dL Creatinine 1.7 mg/dL Urine Protein 2+ WBC 810/hpf RBC none Bacteria none Nitrates none Test of the stool for occult blood is negative. Which of the following is the most appropriate next step in management? A ) Intravenous pyelography B ) Discontinue current medication C ) Antibiotic therapy for recurrent urinary tract infections D ) Insulin therapy for diabetes mellitus E ) Upper endoscopy

20. A 47-year-old woman who is visiting from Australia comes to the physician because of increasing urine output over the past month. She has had

no dysuria or hematuria. She has a history of chronic headaches, peptic ulcer disease, and urinary tract infections. An evaluation 18 months ago for headaches, including CT scan of the head, showed no abnormalities; treatment with ibuprofen and phenacetin was initiated at that time, and her headaches have been well controlled. Her temperature is 37.1 C (98.8 F), blood pressure is 140/82 mm Hg, pulse is 78/min, and respirations are 14/min. Examination shows no abnormalities. Laboratory studies show: Hematocrit 32% Mean corpuscular volume 88 m3 Serum Glucose 130 mg/dL Creatinine 1.7 mg/dL Urine Protein 2+ WBC 810/hpf RBC none Bacteria none Nitrates none Test of the stool for occult blood is negative. Which of the following is the most appropriate next step in management? A ) Intravenous pyelography B ) Discontinue current medication C ) Antibiotic therapy for recurrent urinary tract infections D ) Insulin therapy for diabetes mellitus E ) Upper endoscopy

21. A previously healthy 85-year-old man has had abdominal distention, decreased caliber of stools, and decreased appetite over the past 2 weeks and a 9-kg (20-lb) weight loss over the past 3 months. On sigmoidoscopy, he is found to have a constricting adenocarcinoma of the sigmoid colon; imaging studies show three 1-cm metastases to the liver. Which of the following is the most appropriate next

step in management? A ) No treatment B ) Radiation therapy C ) Chemotherapy D ) Combination radiation therapy and chemotherapy E ) Resection of the colon tumor 22. An 8-year-old girl with asthma is brought to the physician 1 week after an acute exacerbation treated with a 5-day taper course of oral prednisone. This was her first asthma attack of the fall season. Medications include an inhaled corticosteroid daily and a bronchodilator metered-dose inhaler as needed. Her last immunizations were at the age of 5 years prior to entering kindergarten. Her temperature is 37 C (98.6 F), pulse is 92/min, and respirations are 28/min. Examination shows end-expiratory wheezing with forced expiration. Administration of which of the following vaccines is most appropriate at this visit? A ) Haemophilus influenzae type b B ) Influenza virus C ) Meningococcal D ) 23-Valent pneumococcal E ) Varicella

23. A 27-year-old man is brought to the hospital by family members because he has remained in his room for 3 days. He has refused to go to work or eat with the family, and he expresses concern that family or

friends may try to kill him. One week ago, he was despondent when his girlfriend of 5 years abruptly ended their relationship. He has no history of psychiatric illness. Which of the following is the most likely diagnosis? A ) Adjustment disorder with depressed mood B ) Bipolar disorder C ) Brief psychotic disorder D ) Dysthymic disorder E ) Schizoaffective disorder 24. A 35-year-old primigravid woman at 20 weeks' gestation comes to the physician because of vaginal pressure and a watery, pink vaginal discharge for 1 day. Her temperature is 37.5 C (99.5 F). The uterus is palpated at the umbilicus. Fetal heart rate is 140/min. Speculum examination shows that the upper vagina is filled with bulging, shiny, smooth membranes. The cervix cannot be palpated. Which of the following is the most likely mechanism for these findings? A ) Abruptio placentae B ) Cervical incompetence C ) Premature labor D ) Uterine anomaly E ) Uterine infection 25. A previously healthy 45-year-old woman has had fever and progressive confusion over the past 2 days. She is now unable to perform activities of daily living. Her temperature is 38 C (100.4 F). She is oriented only to person. There is no rash, and the neck is supple. A CT

scan of the head shows normal findings. Cerebrospinal fluid analysis shows: Leukocyte count 20/mm3 Glucose 45 mg/dL Protein 110 mg/dL Erythrocyte count 1000/mm3 Which of the following is the most likely diagnosis? A ) Bacterial meningitis B ) Acute alcohol intoxication C ) Brain stem infarction D ) Cerebral infarction E ) Cryptococcal meningitis F ) Enterovirus infection G ) Hepatic encephalopathy H ) Herpes simplex encephalitis I ) Huntington's disease J ) Hypoglycemia K ) Lyme disease 26. A 57-year-old man with multiple myeloma comes to the physician because of a 12-hour history of fever, sharp chest pain with deep inspiration, and cough productive of blood-tinged sputum. His temperature is 38.3 C (101 F), blood pressure is 120/78 mm Hg, pulse is 112/min, and respirations are 28/min. Crackles are heard at the right lung base. His hemoglobin level is 9.2 g/dL, leukocyte count is 2600/mm3, and platelet count is 96,000/mm3. Empiric antibiotics should be directed against which of the following organisms?

A ) Listeria monocytogenes B ) Neisseria meningitidis C ) Pseudomonas aeruginosa D ) Streptococcus bovis E ) Streptococcus pneumoniae 27. A 19-year-old primigravid woman is brought to the emergency department because of a 4-hour history of heavy vaginal bleeding. She has vomited daily for the past month. Her last menstrual period was 15 weeks ago. She has not received prenatal care. She takes no medications. Her temperature is 37 C (98.6 F), blood pressure is 140/90 mm Hg, pulse is 80/min, and respirations are 20/min. Abdominal examination shows a uterus consistent in size with a 20-week gestation with no adnexal masses or tenderness. There is pedal edema. A serum pregnancy test is positive. Urinalysis shows 1+ protein. Which of the following is the most likely cause of this patient's vaginal bleeding? A ) Abruptio placentae B ) Ectopic pregnancy C ) Hydatidiform mole D ) Hyperthyroidism E ) Preeclampsia 28. A 40-year-old man has the sudden onset of excruciating head and neck pain while carrying books from the basement to the attic. His temperature is 37 C (98.6 F), blood pressure is 130/90 mm Hg, and pulse is 90/min. He has photophobia and develops eye pain with lateral eye movements. His neck is markedly stiff and cannot be passively flexed. He

has diffuse hyperreflexia in all extremities with normal strength and sensation. Plantar reflexes are flexor bilaterally. Which of the following is the most likely diagnosis? A ) Cervical osteoarthritis B ) Meningitis C ) Ruptured cervical disc D ) Subarachnoid hemorrhage E ) Syringomyelia

29. A 59-year-old woman comes to the emergency department because of shortness of breath for 2 days. She had stage II breast cancer 5 years ago treated with lumpectomy, radiation, and chemotherapy. Her temperature is 37.5 C (99.5 F), blood pressure is 90/60 mm Hg, and respirations are 24/min. Examination shows jugular venous distention. Heart sounds are distant. Which of the following is the most appropriate next step in management? A ) Antibiotic therapy B ) Anticoagulant therapy C ) Intravenous digoxin therapy D ) Intravenous furosemide therapy E ) Chemotherapy F ) Radiation therapy G ) Pericardiocentesis 30. A 57-year-old man comes for a routine follow-up examination. He has a 10-year history of an intermittent facial rash.

He has been taking propranolol for 2 months for hypertension. Examination shows several erythematous pustules and papules involving the nose and central face. There are telangiectasias at the base of the papules. Which of the following is the most likely explanation for these findings? A ) Acne rosacea B ) Acne vulgaris C ) Basal cell carcinoma D ) Discoid lupus erythematosus E ) Seborrheic dermatitis

31. A 42-year-old man comes to the physician because of malaise, muscle and joint pain, and temperatures to 38.4 C (101.1 F) for 3 days. Three months ago, he underwent cadaveric renal transplantation resulting in immediate kidney function. At the time of discharge, his serum creatinine level was 0.8 mg/dL. He is receiving cyclosporine and corticosteroids. Examination shows no abnormalities. His leukocyte count is 2700/mm3, and serum creatinine level is 1.6 mg/dL; serum cyclosporine level is in the therapeutic range. A biopsy of the transplanted kidney shows intracellular inclusion bodies. Which of the following is the most appropriate next step in management? A ) Increase the dosage of corticosteroids B ) Increase the dosage of cyclosporine C ) Begin amphotericin therapy D ) Begin ganciclovir therapy E ) Begin heparin therapy

32. A 5-month-old boy is brought to the physician because of a 24-hour history of fever, cough, noisy breathing, and difficulty feeding. His symptoms began 3 days ago with nasal discharge, mild cough, and chest congestion. He appears somewhat irritable and is crying. His temperature is 38.5 C (101.3 F), pulse is 108/min, and respirations are 32/min and shallow with a prolonged expiratory phase. On examination, the throat appears normal. A few small anterior and posterior cervical nodes are palpable. Both eardrums are pink but have normal landmarks and mobility. There is good air entry with diffuse bilateral expiratory wheezes on auscultation. An x-ray film of the chest shows hyperinflation. Which of the following is the most likely pathogen? A ) Adenovirus B ) Haemophilus influenzae C ) Mycoplasma pneumoniae D ) Respiratory syncytial virus E ) Streptococcus pneumoniae 3. Three days after undergoing a right hip replacement for rheumatoid arthritis, a 77-year-old man is brought to the physician because of a 2-day history of pain, burning, and itching of his left eye and left side of his forehead. He has the sensation that there is a speck of dirt in his left eye. Current medications include prednisone and methotrexate. Examination of the left eye shows conjunctival injection and swelling of the upper eyelid. There is an erythematous rash over the left side of the forehead and tenderness to palpation from the upper eyelid to the vertex. A photograph of the rash is shown. Which of the following is the most appropriate next step in management? A ) Measurement of erythrocyte sedimentation rate

B ) MRI of the brain with contrast C ) Acyclovir therapy D ) Corticosteroid therapy E ) Lumbar puncture 34. A 54-year-old man with chronic obstructive pulmonary disease undergoes a total hip arthroplasty for avascular necrosis of the femoral head. On the second postoperative day, he has diffuse, profound weakness and vomiting. His blood pressure is 85/50 mm Hg, and pulse is 100/min. The operative site is clean and dry, with minimal output from the drains. Hemoglobin level is 13.8 g/dL, serum sodium level is 132 mEq/L, and serum potassium level is 5.8 mEq/L. Which of the following is most likely to confirm the diagnosis? A ) Measurement of serum thyroid-stimulating hormone level B ) Direct antiglobulin (Coombs') test C ) ACTH stimulation test D ) Ventilation-perfusion lung scans E ) Echocardiography 35. A previously healthy 47-year-old nulliparous woman is brought to the emergency department by ambulance because of acute low back pain radiating to the right posterior leg for 2 hours. The pain began when she bent over at work to retrieve a file from the lowest drawer of a filing cabinet. She does not smoke cigarettes or drink alcohol. Examination shows right paraspinous muscle spasm and pain in the lower back with right straight-leg raising at 30 degrees. She says that she plans to file a claim for a work-related injury. Which of the

following findings is the strongest risk factor for a prolonged episode of pain in this patient? A ) Arrival for care in an ambulance B ) Claim that pain is work-related C ) Gender D ) Nulliparity E ) Positive straight-leg raising test F ) Radiation of the pain into the posterior lower extremity

36. A healthy 27-year-old woman comes to the physician for an annual examination. She is concerned about her risk for an abnormal Pap smear in the future. A history of use of which of the following would increase her risk for cervical cancer? A ) Alcohol B ) Cervical cap C ) Cigarettes D ) Isotretinoin E ) IUD 37. A 72-year-old man is brought to the physician by his daughter because of painless jaundice for 1 month. His wife died 10 years ago, and his daughter is his only child. Before examining the patient, the daughter asks to speak privately with the physician and asks that she be given the results of any tests. She specifically requests that he not be given any "bad news." The patient is alert. His vital signs are

within normal limits. Examination shows scleral icterus and jaundice. There is mild abdominal tenderness on palpation. He is oriented to person, place, and time. A CT scan of the abdomen shows a pancreatic mass with bile duct obstruction and probable metastatic lesions in the liver. Which of the following is the most appropriate next step? A ) Abide by the daughter's wishes B ) Ask the patient if he wishes to discuss his test results, preferably with his daughter present C ) Tell the daughter it is a legal requirement to tell the patient any and all results of medical testing D ) Consult with the hospital attorney E ) Ask another physician to take over the patient's care

38. A 66-year-old man has had numbness and tingling in the hands and feet for 2 weeks. He lives in a homeless shelter and is well fed. He has been treated for pulmonary tuberculosis for 4 months with isoniazid, rifampin, ethambutol, and pyrazinamide. He is compliant with his medication regimen but continues to abuse alcohol. His temperature is 37 C (98.6 F), blood pressure is 136/76 mm Hg, pulse is 72/min, and respirations are 20/min. He is well nourished but depressed and irritable. There is decreased sensation to pain and touch in the hands and feet in a stocking-glove distribution. Which of the following is the most likely nutritional deficiency? A ) Folic acid B ) Niacin C ) Vitamin A

D ) Vitamin B1 (thiamine) E ) Vitamin B2 (riboflavin) F ) Vitamin B6 G ) Vitamin B12 (cyanocobalamin) H ) Vitamin C I ) Vitamin D J ) Vitamin E K ) Vitamin K

39. A 63-year-old man is brought to the emergency department 3 hours after the acute onset of severe right-sided flank pain. He has a 9-year history of gout. His blood pressure is 110/84 mm Hg, pulse is 78/min, and respirations are 16/min. Examination shows normal bowel sounds and no abdominal tenderness or masses. Urinalysis shows 40 erythrocytes/hpf. Intravenous pyelography confirms a right ureteral calculus. Which of the following is the most likely underlying mechanism of this patient's urolithiasis? A ) An increase in urinary pH B ) Damage to the epithelial lining of the ureters C ) Lack of inhibitors of crystal formation D ) Presence of urease-splitting bacteria E ) Urinary supersaturation with uric acid 40. A 27-year-old nulligravid woman comes to the physician for preconceptional counseling. She has a mechanical

mitral heart valve and chronic rheumatoid arthritis. Her cardiac status is New York Heart Association Class II. She feels well. Current daily medications include warfarin, prednisone, and acetaminophen with codeine. Examination shows no abnormalities except for audible clicking from the heart valve. Which of the following is the most appropriate advice for this patient? A ) Chemical dependency counseling before pregnancy B ) Discontinuation of anticoagulant therapy during pregnancy C ) Discontinuation of prednisone during pregnancy D ) Switching from warfarin to heparin before pregnancy E ) No change in treatment before or during pregnancy

41. A 60-year-old man comes to the physician because of difficulty sleeping and concentrating and a 5-kg (10-lb) weight loss over the past 3 months. He also has become withdrawn. He has had chronic pain since sustaining fractures of the left lower extremity, pelvis, and several ribs in a motor vehicle collision 2 years ago. He has a previous history of alcohol abuse. He takes a nonsteroidal anti-inflammatory drug. Which of the following is the most appropriate pharmacotherapy? A ) Carbamazepine B ) Chlordiazepoxide C ) Disulfiram D ) Lithium carbonate E ) Nortriptyline

42. An 82-year-old man comes to the physician because of a 3-day history of low back pain that radiates to the right leg. He also has had a lesion over the right shin and weakness of the right foot. He began taking prednisone 2 weeks ago for acute bronchitis. He has chronic obstructive pulmonary disease, benign prostatic hypertrophy, and glaucoma. Examination shows numerous papular and vesicular lesions over the right anterior and posterior shin. There is weakness of right knee flexion, ankle dorsiflexion, plantar flexion, eversion, and inversion; the right ankle reflex is absent. Sensation to pinprick and cold is decreased over the right lower extremity. Which of the following is the most likely causal organism? A ) Borrelia burgdorferi B ) Epstein-Barr virus C ) Herpes simplex virus 1 D ) Poliovirus E ) Treponema pallidum F ) Varicella-zoster virus

43. A 30-year-old woman comes to the physician because of long-standing unhappiness that may have started when she was rejected by her classmates as a teenager. She says that although she has good days, many days are dominated by negative thoughts about herself. She appears somewhat sad and tends to be readily critical of herself. Although she sleeps satisfactorily, she often finds her energy level decreased by the end of the day. She also has been forgetful. She weighs 59 kg (130 lb) and is 157 cm (62 in) tall. Physical examination and laboratory studies show no abnormalities. Which of the following is the most likely diagnosis?

A ) Adjustment disorder with depressed mood B ) Depersonalization disorder C ) Dissociative identity disorder D ) Dysthymic disorder E ) Hypothyroidism F ) Major depressive disorder G ) Schizoaffective disorder For each patient with cough, select the most appropriate next step in diagnosis. A ) Measurement of serum a1-antitrypsin level B ) Methacholine challenge test C ) Quantitative measurement of serum antibody levels D ) Sweat chloride test E ) Ventilation-perfusion lung scans 44. A 27-year-old woman comes to the physician because of intermittent episodes of shortness of breath and cough over the past 4 months. She says that cold weather and exercise can precipitate her symptoms. Examination shows no abnormalities. The lungs are clear to auscultation and percussion. An x-ray film of the chest and spirometry show no abnormalities. A 32-year-old woman comes to the physician because of a 4-month history of fatigue, cough, and shortness of breath with exertion. She has had two episodes of pneumonia and one episode of severe sinusitis over the past 2 years. She has never smoked. She takes no

medications. Crackles are heard at the left lung base. An x-ray film of the chest shows a left lower lobe infiltrate and scarring of the right base. 46. A 32-year-old man is brought to the emergency department 30 minutes after being involved in a motor vehicle collision. He was the restrained driver. On arrival, he is alert and has shortness of breath. His blood pressure is 80/50 mm Hg, pulse is 130/min, and respirations are 30/min. Examination shows jugular venous distention and abrasions over the left hemithorax. The trachea is deviated to the right. Breath sounds are absent on the left. Which of the following is the most likely cause of the hypotension? A ) Cardiogenic shock B ) Congestive heart failure C ) Decreased systemic vascular resistance D ) Decreased venous return E ) Hypovolemic hypoperfusion F ) Increased systemic vascular resistance BLOCK 4:-1. A previously healthy 2-year-old boy is brought to the physician 20 minutes after an episode of cyanosis and loss of consciousness that lasted 3 minutes. The symptoms occurred after his mother scolded him for climbing onto the dining room table. The mother says that the child began to cry, let out a deep sigh, stopped breathing, and jerked his arms and legs back and forth. On arrival, he is alert and active. Neurologic examination shows no focal findings. Which of the following is the most appropriate next step in management? A ) Reassurance

B ) Electroencephalography C ) CT scan of the head D ) Anticonvulsant therapy E ) Lumbar puncture

2. A case-control study is conducted to assess the risk for intussusception in infants under the age of 1 year who receive the rotavirus vaccine. The medical records of all those who received the vaccine and those who did not receive the vaccine over a 6-month period are reviewed. Results show 125 cases per 100,000 infant-years for infants who received the vaccine compared to 45 cases per 100,000 infant-years for infants who did not receive the vaccine. The investigators conclude that the relative risk for intussusception is 1.9 times greater in infants who receive the rotavirus vaccine (95% confidence interval of 0.57.7 and p=0.39). Which of the following is the most accurate interpretation of these results? A ) The results do not show an association between rotavirus vaccine and intussusception, but they may be related B ) The results show sufficient statistical power to identify an association between rotavirus vaccine and intussusception C ) Rotavirus vaccine is associated with a 39% risk for intussusception D ) Rotavirus vaccine causes intussusception in 1.9% of infants E ) Rotavirus vaccine prevents 80 cases of intussusception per 100,000 infant-years

3. A 42-year-old woman, gravida 3, para 3, comes to the emergency department 24 hours after the onset of moderate epigastric pain radiating to the back. Her last menstrual period was 3 weeks ago. She has no history of serious illness and has never undergone an operative procedure. She weighs 72 kg (160 lb) and is 157 cm (62 in) tall. Her temperature is 37.2 C (99 F), blood pressure is 130/90 mm Hg, and pulse is 100/min. Abdominal examination shows mild distention, epigastric tenderness, and voluntary guarding. Test of the stool for occult blood is negative. Laboratory studies show: Hematocrit 44% Leukocyte count 12,000/mm3 Serum Na+ 138 mEq/L Cl 100 mEq/L K+ 4 mEq/L HCO3 25 mEq/L Bilirubin, total 1.6 mg/dL Alkaline phosphatase 100 U/L Aspartate aminotransferase (AST, GOT) 14 U/L Alanine aminotransferase (ALT, GPT) 12 U/L Amylase 1100 U/L Ultrasonography shows gallstones; the gallbladder wall is 1 mm and the common bile duct is 5 mm in diameter. Which of the following is the most likely diagnosis? A ) Acute cholecystitis B ) Acute pancreatitis C ) Acute perihepatitis D ) Ascending cholangitis E ) Duodenal ulcer F ) Viral hepatitis

4. A 13-year-old girl is brought to the emergency department because of shortness of breath for 2 hours. The symptoms began after consuming chili, cornbread, and fruit salad with strawberries, kiwi, and bananas. She has a 1-year history of shortness of breath while playing soccer or baseball and uses a bronchodilator inhaler as needed while exercising. She is allergic to penicillin and pineapples. Her blood pressure is 80/60 mm Hg, pulse is 120/min and regular, and respirations are 20/min with use of accessory muscles. Examination of the lungs shows poor air entry bilaterally with diffuse expiratory wheezes. Which of the following is the most appropriate initial pharmacotherapy? A ) Inhaled bronchodilators B ) Inhaled cromolyn sodium C ) Inhaled ipratropium bromide D ) Intravenous corticosteroids E ) Subcutaneous epinephrine 5. A 52-year-old woman comes to the emergency department 6 days after knee arthroplasty because of constant, right-sided chest pain and shortness of breath for 24 hours. Her blood pressure is 110/50 mm Hg, pulse is 114/min, and respirations are 24/min. Examination of the heart, lungs, and extremities shows no abnormalities. Arterial blood gas analysis on room air shows: pH 7.49 PCO2 29 mm Hg PO2 66 mm Hg Ventilation-perfusion lung scans show a low probability for pulmonary embolus. An ECG shows sinus tachycardia; an x-ray film of the chest shows no abnormalities. After the evaluation, the patient is pain-free

and wishes to go home. Which of the following is the most appropriate next step in management? A ) Discharge home and reexamination in 2 weeks B ) Exercise stress test C ) Pulmonary function tests D ) Echocardiography E ) Pulmonary angiography F ) Ibuprofen therapy 6. A 25-year-old woman comes to the physician because of "spells" characterized by sweating, palpitations, and shortness of breath that have awakened her at night several times over the past 3 months. She resigned from her job as a sales clerk 6 months ago and now works from home as a telemarketer. She recently began going to the grocery store late at night because she is too nervous around people. She says that she has been feeling sad lately. On mental status examination, she is fully oriented, and her range of affect is full. Which of the following is the most likely diagnosis? A ) Adjustment disorder with anxiety B ) Adjustment disorder with depressed mood C ) Dysthymic disorder D ) Generalized anxiety disorder E ) Major depressive disorder F ) Panic disorder with agoraphobia G ) Post-traumatic stress disorder

7. A 65-year-old woman has a 6-month history of progressive irritability, palpitations, heat intolerance, frequent bowel movements, and a 6.8-kg (15-lb) weight loss. She has had a neck mass for more than 10 years. 131I scan shows an enlarged thyroid gland with multiple areas of increased and decreased uptake. Which of the following is the most likely diagnosis? A ) Defect in thyroxine (T4) biosynthesis B ) Graves' disease C ) Multinodular goiter D ) Riedel's thyroiditis E ) Thyroid carcinoma F ) Thyroiditis G ) Toxic adenoma H ) Triiodothyronine (T3) thyrotoxicosis A ) Acute gastrointestinal bleeding B ) Adrenal insufficiency C ) Aortic valve rupture D ) Cardiac tamponade E ) Congestive heart failure F ) Pneumonia G ) Pulmonary embolism H ) Sepsis

8. A previously healthy 62-year-old man is brought to the emergency department by paramedics 40 minutes after the sudden onset of severe shortness of breath while dressing this morning. He is unable to provide additional medical history. He is in severe respiratory distress. His temperature is 37.8 C (100 F), blood pressure is 90/60 mm Hg, pulse is 120/min and regular, and respirations are 24/min. Examination shows marked jugular venous distention. The lungs are clear to auscultation. Cardiac examination shows a nondisplaced and discrete point of maximal impulse and normal S1 and S2; there is an S4 and a right parasternal heave. Abdominal examination shows no abnormalities. There is no edema of the lower extremities. Laboratory studies show: Hematocrit 40% Leukocyte count 14,000/mm3 Platelet count 350,000/mm3 Arterial blood gas analysis on 5 L/min of oxygen: pH 7.5 PCO2 16 mm Hg PO2 64 mm Hg A previously healthy 67-year-old woman is brought to the emergency department by paramedics 40 minutes after the sudden onset of shortness of breath while shopping. She is unable to provide additional medical history. She is in severe respiratory distress. Her temperature is 37 C (98.6 F), blood pressure is 90/60 mm Hg, pulse is 120/min and regular, and respirations are 24/min. Examination shows marked jugular venous distention. Diffuse crackles are heard throughout all lung fields. Cardiac examination shows an enlarged point of maximal impulse and normal S1 and S2; there is an S3. Abdominal examination shows no abnormalities. There is no edema of the lower extremities. Laboratory studies show: Hematocrit 38% Leukocyte count 12,000/mm3

Platelet count 350,000/mm3 Arterial blood gas analysis on 5 L/min of oxygen: pH 7.5 PCO2 16 mm Hg PO2 64 mm Hg

10. A mentally competent 76-year-old man is in the terminal stage of multiple myeloma. He is unable to move and requires 24-hour nursing care. Increasing doses of narcotics are needed to control severe pain. As a result, when he is pain-free, respiratory function is impaired and consciousness is clouded. The patient says he cannot live with this degree of pain and asks to be given a lethal injection of pain medication. Which of the following is the most appropriate step regarding the pain medication? A ) Reduce the dosage so as not to impair respiration B ) Administer the dosage necessary to control pain despite respiratory impairment C ) Administer the dosage necessary to control pain and add a centrally acting stimulant D ) Appeal to the family to convince the patient to tolerate a bit more pain

11. A 2-day-old newborn is brought to the physician because of a generalized rash for 6 hours. The newborn is active, alert, and feeding well. His temperature is 36.9 C (98.4 F). Examination shows a rash consisting of numerous white and pale yellow papules with a large base of macular erythema over the trunk and extremities. Wright's stain of scrapings from the lesions shows eosinophils. Which of the following is

the most appropriate next step in management? A ) Reassurance B ) Topical corticosteroid therapy C ) Intravenous acyclovir therapy D ) Intravenous ampicillin and gentamicin therapy E ) Intravenous nafcillin therapy

12. A 4-year-old boy is brought for a well-child examination. He uses two-word phrases, can say his first name but not his last name, and cannot identify colors. He is just beginning toilet training. His 7-year-old sister has a learning disability and attends special education classes. Genital development is Tanner stage 1; testes are large. Which of the following is the most appropriate next step in diagnosis? A ) Reexamination in 6 months B ) Thyroid function tests C ) DNA testing D ) Measurement of bone age E ) CT scan of the head

3. A 72-year-old man is brought to the physician because of a 2-day history of nausea and vomiting. The vomitus has been clear, and no blood has been noted. He has had a decreased appetite for the past week. There is no associated pain or altered bowel function. He reports that he is not seeing things correctly. He takes warfarin and digoxin for atrial fibrillation, hydrochlorothiazide for hypertension, and potassium supplements that he discontinued 3 weeks ago when he

ran out of tablets. His temperature is 37 C (98.6 F), blood pressure is 144/88 mm Hg, and pulse is 52/min and irregular. Bowel sounds are normal. The abdomen is soft and nontender without rebound or guarding. No organomegaly or masses are palpated. Which of the following is the most likely diagnosis? A ) Acute pancreatitis B ) Brain tumor C ) Diabetic gastroparesis D ) Diabetic ketoacidosis E ) Drug toxicity F ) Food poisoning G ) Gastric bezoar H ) Gastric carcinoma I ) Pyloric channel ulcer J ) Small-bowel obstruction K ) Uremia 14. A 27-year-old primigravid woman at 12 weeks' gestation comes to the emergency department 2 hours after the sudden onset of bright red vaginal bleeding. She has not had abdominal cramping. Pelvic examination shows a small amount of brownish blood in the posterior fornix of the vagina. The cervix is closed. The uterus is palpable 3 cm above the pelvic brim. Fetal heart tones are easily audible at 167/min by Doppler. Which of the following is the most likely diagnosis? A ) Abruptio placentae

B ) Ectopic pregnancy C ) Incomplete abortion D ) Placenta previa E ) Threatened abortion F ) Normal pregnancy 15. A 72-year-old man is brought to the physician by his son because of a 4-day history of increasing confusion and memory problems. The son says that his father's ability to function independently has been generally declining over the past few years, and he has become much more impaired over the past week. The patient has had at least three to four previous episodes of a sudden decline of cognitive functioning over the past 3 years without full recovery. He has a history of hypertension. His blood pressure is 160/95 mm Hg without orthostatic changes. Neurologic examination shows no focal findings. Mini-Mental State Examination score is 21/30. Which of the following is the most likely underlying pathophysiologic process? A ) Central nervous system demyelination B ) Central nervous system infection C ) Diffuse axonal injury D ) Diffuse cortical atrophy E ) Left temporal lobe infarction F ) Multiple, small, central nervous system infarctions G ) Subdural hematoma

16. A 72-year-old woman comes for a routine health maintenance examination. She has a 3-year history of occasional loss of small amounts of urine when she coughs or sneezes. She has had no pain or burning with urination. She has hypertension treated with daily hydrochlorothiazide. She underwent an appendectomy at the age of 10 years. She has three children and had uncomplicated pregnancies. Examination shows no abnormalities except for a moderate cystocele. Which of the following is the most likely cause of this patient's urinary symptoms? A ) Chronic infectious trigonitis B ) Large intravesical calculus C ) Obstetric trauma D ) Polycystic kidney disease E ) Spastic neurogenic bladder 17. A 47-year-old woman is brought to the emergency department by her husband because of hallucinations and agitation for 6 hours. She has a 10-year history of alcoholism. Her last alcoholic drink was 48 hours ago. She is agitated and inattentive. Her temperature is 38.3 C (101 F), blood pressure is 190/120 mm Hg, and pulse is 110/min. She is oriented to person but not to place or time. During the examination, she shrieks, "Make the lizards go away." Which of the following is the most appropriate initial step in management? A ) Electroencephalography B ) CT scan of the head C ) Intravenous ampicillin therapy D ) Intravenous haloperidol therapy

E ) Intravenous lorazepam therapy 18. A 27-year-old woman comes to the physician because of a 3-week history of fever, night sweats, rash on both legs, nonproductive cough, and pain and swelling in her wrists and knees. She has not had weight loss. Her temperature is 37.7 C (99.8 F), blood pressure is 110/70 mm Hg, pulse is 96/min, and respirations are 14/min. The lungs are clear to auscultation. Cardiac examination shows no abnormalities. There is swelling and warmth over the wrists and knees bilaterally and tender red nodules on the anterior surface of both lower extremities. An x-ray film of the chest shows bilateral hilar fullness. Which of the following is the most likely diagnosis? A ) Carcinoma of the lung B ) Histoplasmosis C ) Hodgkin's disease D ) Sarcoidosis E ) Tuberculosis 19. A 52-year-old woman comes to the physician because of difficulty climbing stairs for 4 months. She has also noted that her thighs hurt when she presses on them. She has had increasing difficulty combing her hair because she tires easily. On examination, she pushes herself out of the chair with her arms. There is weakness of the proximal muscles of the extremities. Which of the following is the most likely diagnosis? A ) Cauda equina syndrome B ) Cerebellar degeneration C ) Cervical spinal cord compression

D ) Diabetic polyneuropathy E ) Femoral artery insufficiency F ) Guillain-Barr syndrome G ) Lumbar spinal stenosis H ) Multiple sclerosis I ) Normal-pressure hydrocephalus J ) Polymyositis K ) Sensory neuropathy L ) Tabes dorsalis

20. A 72-year-old man comes to the physician because of generalized weakness and night sweats for 6 months. During this period he has had a 5-kg (11-lb) weight loss. He has had polycythemia vera for 12 years treated with hydroxyurea and multiple phlebotomies. Examination shows cachexia. The liver is enlarged and nontender with a span of 13 cm; the spleen is enlarged. Hematocrit is 27%, leukocyte count is 3200/mm3, and platelet count is 150,000/mm3. A blood smear is shown. Which of the following is the most likely diagnosis? A ) Acute myelogenous leukemia B ) Cirrhosis of the liver C ) Hodgkin's disease D ) Miliary tuberculosis E ) Myelofibrosis

21. A 30-year-old man has had increasingly severe low back pain since lifting a heavy object at work 3 days ago. The pain does not radiate and is not associated with bowel or bladder problems. He has a history of occasional stiffness and mild pain in his lower back. There is bilateral paravertebral muscle tenderness in the lumbar region and limited flexion of the lumbosacral spine. Examination of the lower extremities shows normal muscle strength and sensation; straight-leg raising is negative bilaterally. Deep tendon reflexes at the knees and ankles are normal. Which of the following is the most appropriate next step in management? A ) Exercises to strengthen abdominal muscles B ) Exercises to strengthen paravertebral muscles C ) Bed rest for 5 to 7 days D ) Use of a muscle relaxant E ) Use of a nonsteroidal anti-inflammatory drug

22. A 42-year-old woman, gravida 2, para 2, has had increasing fatigue, dyspnea, orthopnea, and paroxysmal nocturnal dyspnea over the past 2 days. She has had several episodes of hemoptysis; she had one episode of pulmonary edema during pregnancy 2 years ago. A loud S1, a snapping sound in diastole, and a rumbling diastolic murmur are heard at the apex. Which of the following is the most likely cause of her condition? A ) Atrial myxoma B ) Bicuspid aortic valve C ) Postpartum cardiomyopathy

D ) Rheumatic heart disease E ) Viral myocarditis 23. A 37-year-old woman has had unilateral breast pain, fever, and chills for 24 hours. She breast-feeds her 1-month-old newborn. Her temperature is 38 C (100.4 F). Examination shows swelling, erythema, and localized tenderness of the left breast. Which of the following is the most appropriate next step in management? A ) Application of ice packs to the affected breast B ) Use of a breast pump C ) Immediate discontinuation of breast-feeding D ) Bromocriptine therapy E ) Penicillinase-resistant antibiotic therapy 24. A 19-year-old woman, gravida 2, para 1, at 39 weeks' gestation is admitted in labor. Contractions occur every 2 to 3 minutes. The cervix is 4 cm dilated and 80% effaced. She requests an epidural for pain control. Ten minutes after the epidural is administered, she becomes nauseated and diaphoretic and vomits. Her blood pressure is 60/palpable mm Hg. A fetal heart tracing shows sustained fetal decelerations. The cervix is now 8 cm dilated. The most appropriate next step in management is administration of which of the following? A ) Ephedrine B ) Magnesium sulfate C ) Nifedipine D ) Oxytocin

E ) Terbutaline 25. An asymptomatic 47-year-old man comes for a routine follow-up examination 3 weeks after sustaining an inferior wall myocardial infarction. His recovery was complicated by transient premature ventricular contractions during the first 2 days of hospitalization. An exercise stress test prior to discharge showed no pain at 70% of his predicted maximum exercise capacity. Medications include daily aspirin and pravastatin. His blood pressure is 136/80 mm Hg, pulse is 80/min and regular, and respirations are 16/min. Cardiopulmonary examination shows no abnormalities. Which of the following is the most appropriate additional pharmacotherapy for this patient? A ) Clonidine B ) Metoprolol C ) Quinidine D ) Sulfinpyrazone E ) Verapamil

26. A 35-year-old woman is brought to the emergency department by her family because of shortness of breath, tightness in her chest, and palpitations for 2 hours. Over the past 11 months, she has had five similar episodes; during the last episode 3 weeks ago, she was treated with an intravenous medication that caused conversion to sinus rhythm. Her blood pressure is 95/60 mm Hg, and pulse is 165/min and regular. The lungs are clear to auscultation. Which of the following is the most likely underlying dysrhythmia? A ) Accelerated idioventricular rhythm B

) Accelerated junctional rhythm C ) Atrial fibrillation D ) Multifocal atrial tachycardia E ) Normal sinus rhythm F ) Paroxysmal supraventricular tachycardia G ) Premature supraventricular beats H ) Premature ventricular beats I ) Sick sinus syndrome J ) Sinus bradycardia K ) Sinus tachycardia L ) Ventricular fibrillation M ) Ventricular tachycardia

27. A 4-year-old boy is brought to the emergency department because of a 1-day history of fever and increasing difficulty breathing. Over the past year, he has had recurrent bacterial infections including cervical lymphadenitis, septic arthritis, and pneumonia. His temperature is 38.9 C (102 F), blood pressure is 80/60 mm Hg, pulse is 115/min, and respirations are 38/min and labored. Breath sounds are decreased over the left anterior chest. Scattered crackles are heard on auscultation. A thoracentesis shows purulent fluid. Laboratory studies show: Hematocrit 36% Leukocyte count 18,000/mm3 Segmented neutrophils 85%

Lymphocytes 15% Platelet count 200,000/mm3 Pleural fluid Leukocyte count 75,000/mm3 Segmented neutrophils 98% Lymphocytes 2% Nitroblue tetrazolium test is abnormal. A Gram's stain of the pleural fluid shows numerous gram-positive cocci in the segmented neutrophils. An x-ray film of the chest shows left lower lobe pneumonia with pleural effusion. The most likely cause of this patient's symptoms is a defect of which of the following? A ) Chemotaxis B ) Immotile cilia C ) Opsonization D ) Phagocytic oxidative metabolism E ) Phagocytosis F ) T-lymphocyte function

28. A 59-year-old woman comes to the emergency department 45 minutes after the onset of chest discomfort that is not relieved by three nitroglycerin tablets. Over the past 3 months, she has had similar episodes characterized by nonradiating pain and a feeling of heaviness; the episodes were exacerbated by exertion or heavy meals and were slowly relieved by rest. Sublingual nitroglycerin has provided rapid relief of symptoms in the past. She has hypercholesterolemia, type 2 diabetes mellitus, and peptic ulcer disease. She smoked two packs of cigarettes daily for 25 years but quit 5 years ago. She appears anxious and diaphoretic and is nauseated. Examination shows no other abnormalities except for an S4. Which of the following is the most likely diagnosis? A ) Acute aortic dissection

B ) Angina pectoris C ) Esophageal spasm D ) Myocardial infarction E ) Pulmonary embolism

29. A 27-year-old woman comes to the physician because of increasingly severe daily headaches over the past 3 months. The headaches are diffuse and often occur at the base of the skull. For the past month, she has had brief episodes of visual darkening when standing. She has been amenorrheic for the past year and has had an 18-kg (40-lb) weight gain during this period. She now weighs 118 kg (260 lb) and is 152 cm (60 in) tall. Funduscopic examination shows papilledema, several flame hemorrhages, and an enlarged blind spot bilaterally. Visual acuity is 20/20 bilaterally. A CT scan of the head shows no abnormalities. Which of the following is the most likely underlying cause of these findings? A ) Cerebral venous occlusion B ) Communicating hydrocephalus C ) Cytotoxic edema D ) Idiopathic intracranial hypertension E ) Impaired absorption of cerebrospinal fluid F ) Infratentorial mass lesion G ) Interstitial edema H ) Overproduction of cerebrospinal fluid I ) Vasogenic edema

30. A 75-year-old man with a 3-year history of progressive cognitive impairment due to dementia, Alzheimer's type, has had nocturnal disorientation for 2 weeks. He lives at home with his wife. He is otherwise healthy and takes no medications. Physical examination shows normal findings. He is disoriented to time and place, has poor short-term memory, is unable to do simple arithmetic, and has a poor understanding of general information. Which of the following is the most appropriate initial step in management? A ) Increase in home nighttime lighting B ) Prescription for chloral hydrate C ) Prescription for diazepam D ) Prescription for haloperidol E ) Use of nighttime mechanical restraints

31. A 62-year-old woman comes to the physician because of severe pain and swelling of her right knee for 1 day. She has no history of joint disease or trauma to the knee. She has hypertension treated with hydrochlorothiazide and type 2 diabetes mellitus treated with glyburide. She is sexually active only with her husband, and they have sexual intercourse one to two times each week. Her temperature is 37 C (98.6 F), blood pressure is 140/84 mm Hg, and pulse is 80/min. Examination of the right knee shows edema, erythema, and exquisite tenderness to light touch; there is an effusion. The remainder of the examination shows no abnormalities. Which of the following is the most likely mechanism of these findings? A ) Immune complex deposition B

) Inflammatory reaction to antisynovial antibodies C ) Inflammatory reaction to monosodium urate crystals D ) Neisseria gonorrhoeae infection E ) Streptococcus pneumoniae infection 32. Two days after a cholecystectomy, a 42-year-old woman has shortness of breath. Her temperature is 37.5 C (99.5 F), blood pressure is 110/70 mm Hg, pulse is 103/min, and respirations are 24/min. There is abdominal tenderness. Examination shows no other abnormalities. An x-ray film of the chest shows minimal linear markings in the right lower lobe. Arterial blood gas analysis on 45% oxygen shows: pH 7.41 PCO2 40 mm Hg PO2 52 mm Hg Which of the following is the most likely explanation for her hypoxia? A ) Acute respiratory distress syndrome B ) Atelectasis C ) Congestive heart failure D ) Fat embolism syndrome E ) Pneumonia F ) Pneumothorax

33. A 37-year-old woman comes to the emergency department because of a 3-day history of increasingly severe abdominal pain, nausea, and vomiting. Twelve years ago, she had a hysterectomy because of severe dysfunctional uterine bleeding. Her temperature is 37 C (98.6 F), blood

pressure is 106/70 mm Hg, pulse is 110/min, and respirations are 12/min. Examination shows a distended, tympanic abdomen with diffuse tenderness but no guarding; bowel sounds are hypoactive. Her leukocyte count is 10,000/mm3, and hematocrit is 44%. An x-ray film of the abdomen is shown. Which of the following is the most appropriate initial step in management? A ) CT scan of the abdomen B ) Intravenous neostigmine therapy C ) Esophagogastroduodenoscopy D ) Nasogastric intubation E ) Laparotomy

34. A healthy 27-year-old woman comes for a routine health maintenance examination. Her blood pressure is 185/90 mm Hg. Examination shows no other abnormalities except for hypertensive retinopathy. Serum studies show a sodium level of 140 mEq/L, potassium level of 4 mEq/L, and creatinine level of 1.1 mg/dL. A complete blood count, serum catecholamine levels, and urinalysis are within normal limits. At two subsequent visits, her blood pressure is 190/100 mm Hg and 182/96 mm Hg, respectively. Which of the following is the most appropriate next step in diagnosis? A ) 24-Hour urine collection for measurement of creatinine clearance B ) 24-Hour urine collection for measurement of 17-hydroxycorticosteroid and total 17-ketosteroid levels C ) Measurement of serum aldosterone level D ) Magnetic resonance angiography of renal vessels

E ) CT scan of the abdomen

35. A 76-year-old man has had fatigue and loss of interest in daily activities over the past 4 months. He sleeps poorly and has had a 4.5-kg (10-lb) weight loss during this period. He states that he has probably lived long enough. His blood pressure is 110/78 mm Hg, and pulse is 68/min. Examination shows a slow return of deep tendon reflexes. Measurement of which of the following serum levels is the most appropriate next step in management? A ) Calcium B ) Creatinine C ) Glucose D ) Testosterone E ) Thyroid-stimulating hormone 36. A 29-year-old woman is brought to the physician by her father. She has been working continuously without sleep for 3 days on a project which she claims "Allah and Jesus have told me to do" that will "unify all knowledge in physics and medicine and philosophy." She hears the voice of her dead brother telling her that she will be "the next Virgin Mary." Her father states that she was treated for depression for 1 year at the age of 19 years. Which of the following is the most likely diagnosis? A ) Bipolar disorder B ) Borderline personality disorder C ) Major depressive disorder D ) Schizophrenia

E ) Substance-induced mood disorder

37. A 67-year-old man is hospitalized for treatment of renal insufficiency. Three days after admission, his pulse is 40/min. An ECG shows tall, tented T waves. Serum studies show a sodium level of 134 mEq/L, potassium level of 6.9 mEq/L, and glucose level of 85 mg/dL. The most appropriate next step in management is intravenous administration of which of the following? A ) Calcium, furosemide, and 3% saline B ) Calcium, insulin, and digitalis C ) Calcium, insulin, and glucose D ) Glucose, furosemide, and phosphate E ) Glucose, glucagon, and bicarbonate 38. Six months after the delivery of her fourth child, a 37-year-old woman undergoes laparoscopic tubal ligation. Menses occur at regular 28-day intervals. During the operation, she is found to have a small dark lesion in the cul-de-sac and filmy adhesions surrounding the ovaries. A biopsy specimen of a cul-de-sac lesion confirms the diagnosis of endometriosis. Which of the following is the most appropriate next step in management? A ) Danazol therapy B ) Gonadotropin-releasing hormone agonist therapy C ) Oral contraceptive therapy D ) Total abdominal hysterectomy and bilateral salpingo-oophorectomy E

) No further treatment indicated A ) Administration of parenteral antibiotics B ) Admission to the hospital for medical management C ) Admission to the hospital for operative management D ) Colon contrast studies E ) Discharge for follow-up by personal physician F ) Endoscopy G ) MRI of the abdomen H ) Observation in the emergency department 39. A previously healthy 6-month-old boy is brought to the emergency department because of a 12-hour history of intermittent episodes of inconsolable crying associated with drawing up of the legs. Over the past 6 hours, he has had intermittent diarrhea that is reddish and mucoid, and for the past 3 hours he has been somnolent. On examination, he is sleepy but arousable. His temperature is 38.1 C (100.6 F), blood pressure is 90/55 mm Hg, pulse is 140/min, and respirations are 38/min. He cries when his abdomen is palpated; a mass is felt in the right lower quadrant. His leukocyte count is 12,400/mm3 (50% segmented neutrophils, 8% bands, 1% eosinophils, 40% lymphocytes, and 1% monocytes). Serum electrolyte levels are within normal limits. An x-ray film of the abdomen shows no free air. A ) Administration of parenteral antibiotics B ) Admission to the hospital for medical management C ) Admission to the hospital for operative management

D ) Colon contrast studies E ) Discharge for follow-up by personal physician F ) Endoscopy G ) MRI of the abdomen H ) Observation in the emergency department 40. A previously healthy 14-year-old girl is brought to the emergency department because of abdominal pain for 12 hours. She has a 1-week history of brownish vaginal discharge. Menarche was at the age of 12 years, and her periods have occurred at regular 28-day intervals over the past year. Her last menstrual period was 7 weeks ago. Her temperature is 37 C (98.6 F), blood pressure is 85/55 mm Hg, pulse is 145/min, and respirations are 24/min. Abdominal examination shows generalized tenderness, and there is guarding with rebound in the right lower quadrant. Her hematocrit is 24%, and leukocyte count is 9400/mm3 (60% segmented neutrophils, 3% bands, 1% eosinophils, 35% lymphocytes, and 1% monocytes). Serum electrolyte levels are within normal limits. 41. A 49-year-old woman is admitted to the hospital because of renal failure. She has had episodes of flank pain over the past 20 years. She has also had nocturia 2 to 3 times nightly for 10 years. Her blood pressure is 160/100 mm Hg. Examination shows pale mucous membranes. A mass is palpated in the right flank. Which of the following is the most likely diagnosis? A ) Horseshoe kidney B ) Nephrolithiasis C ) Papillary necrosis D

) Polycystic kidney disease E ) Renal cell carcinoma 42. On routine annual screening, an asymptomatic 27-year-old man has a positive PPD skin test. One year ago, a PPD skin test was negative. He works as a nurse. Three years ago, he was diagnosed with hepatitis A after a trip to South America. Examination shows no abnormalities. His serum aspartate aminotransferase (AST, GOT) activity is 10 U/L, and serum alanine aminotransferase (ALT, GPT) activity is 14 U/L. An x-ray film of the chest shows no abnormalities. Which of the following is the most appropriate chemoprophylaxis? A ) Isoniazid and folic acid supplementation B ) Isoniazid and rifampin C ) Isoniazid and vitamin B1 (thiamine) supplementation D ) Isoniazid and vitamin B6 supplementation E ) No prophylaxis indicated 43. A 4-month-old boy is brought to the physician because of a 2-day history of fever and progressive redness around his right eye. He has had persistent diarrhea and oral candidiasis since birth and was treated for pneumococcal pneumonia at the age of 2 months. He appears ill. His temperature is 39 C (102.2 F), pulse is 130/min, and respirations are 25/min. Examination shows violaceous preseptal (periorbital) cellulitis and oral candidiasis. Laboratory studies show: Hemoglobin 10 g/dL Leukocyte count 3000/mm3 Segmented neutrophils 85% Lymphocytes 15% Platelet count 350,000/mm3 Serum

IgA <5 mg/dL IgG 300 mg/dL IgM <5 mg/dL Which of the following is the most likely diagnosis? A ) AIDS B ) Chronic granulomatous disease C ) Severe combined immunodeficiency D ) Thymic-parathyroid dysplasia (DiGeorge syndrome) E ) X-linked agammaglobulinemia

44. One week after undergoing an uncomplicated liver transplant for biliary atresia, a 3-year-old boy appears jaundiced. Examination shows scleral icterus. His serum aspartate aminotransferase (AST, GOT) activity has increased to 1300 U/L, and serum alanine aminotransferase (ALT, GPT) activity has increased to 2500 U/L. His serum bilirubin level is 3.5 mg/dL, and serum alkaline phosphatase activity is 100 U/L. Which of the following is the most likely artery responsible for this patient's gastrointestinal symptoms? A ) Hepatic B ) Ileocolic C ) Inferior mesenteric D ) Left gastric E ) Left gastroepiploic F ) Middle colic G ) Posterior penetrating H ) Right colic

I ) Right gastroepiploic J ) Splenic K ) Superior hemorrhoidal L ) Superior mesenteric

45. A 2-year-old girl has had fever and bloody diarrhea for 10 days. A stool culture obtained 7 days ago grew Salmonella species sensitive to amoxicillin. A blood culture was negative. Despite beginning oral amoxicillin therapy 4 days ago, her diarrhea has persisted. Current examination shows no other abnormalities except for a temperature of 38.6 C (101.5 F). Which of the following is the most likely explanation for the failure of amoxicillin to improve her symptoms? A ) Amoxicillin does not alter the course of Salmonella enteritidis B ) Amoxicillin has caused pseudomembranous colitis C ) Amoxicillin is absorbed at the level of the jejunum, leaving no drug to be delivered to the colon D ) Oral amoxicillin is not absorbed into the systemic circulation in the presence of diarrhea E ) Salmonella has expressed an inducible -lactamase that inactivates amoxicillin 46. A 23-year-old man comes to the physician because of a 1-month history of intermittent right-sided abdominal pain, nausea, and vomiting. He has sickle cell disease and has been treated several times for painful crises. Examination of the abdomen shows tenderness to palpation of the

right upper quadrant on inspiration. Laboratory studies show: Hematocrit 25% Leukocyte count 11,000/mm3 Serum Bilirubin Total 3.2 mg/dL Direct 0.3 mg/dL Alkaline phosphatase 56 U/L Ultrasonography of the gallbladder shows a filling defect. Which of the following is the most likely cause of this patient's hyperbilirubinemia? A ) Aggregation of cholesterol in the gallbladder B ) Inhibition of glucuronosyltransferase C ) Lysis of erythrocytes D ) Malnutrition-induced cirrhosis E ) Neoplastic growth in the gallbladder Answers:

http://nbmeanswerkeys.blogspot.com/

Vous aimerez peut-être aussi